Вы находитесь на странице: 1из 41

1

Contents
e. termination of employment .................................................................... 2
ii. effects: valid dismissal; illegal dismissal .................................................. 2
98. Jenny M Agabon, et al. vs NLRC .............................................................. 2
99. JAKA FOOD P]ROCESSING CORP vs PACOT ............................................... 4
iv. preventive suspension; constructive dismissal; suspension of business
operation. .................................................................................................... 5
100. ALU-TUCP v. NLRC .................................................................................. 5
101. Cosmos Bottling Corp., vs NLRC, 255 ...................................................... 6
102. Purefoods Corporation vs. NLRC, et. al ................................................... 7
103. PHILIPPINE FRUIT & VEGETABLE INDUSTRIES, INC. vs. NLRC.................. 9
104. PHILIPS SEMICONDUCTORS (PHILS.), INC. V. FADRIQUELA .................. 10
105. Alcira vs NLRC ........................................................................................ 11
106. Mitsubishi v. crysler .............................................................................. 12
107. PANGILINAN et. al. v. GENERAL MILLING CORP. .................................. 12
108. Ravago v. Esso Eastern Marine, Ltd. ..................................................... 13
109. Hacienda Bino/Hortencia Starke vs. Cuenca ......................................... 14
110. PHILIPPINE GLOBAL COMMUNICATIONS, INC. vs. RICARDO DE VERA . 15
111. Integrated Contractor and Plumbing Works, Inc. vs. NLRC .................. 16
112. LOLITA R. LACUESTA vs. ATENEO DE MANILA UNIVERSITY .................. 18
113. POSEIDON FISHING VS NLRC ................................................................. 19
114. Abesco Construction and Dev. Corp. v Ramirez .................................... 20
115. Cebu Metal v. Saliling, 501 SCRA 61 ...................................................... 21
116. LIGANZA vs RBL SHIPYARD CORPORATION ........................................... 22
117. Fabela, et. al vs. San Miguel Corporation ............................................. 23
118. SORIANO vs. NLRC and PLDT ................................................................. 24
119. Caseres vs. Universal Robina Sugar Milling Corporation (URSUMCO) .. 25
120. Pier 8 Arrastre and Stevedoring Services Inc. vs. Boclot ....................... 26
121. Pacquing et. al. vs. Coca-Cola Phils. ...................................................... 27
122. COCOMANGAS HOTEL BEACH RESORT and/or MUNRO v. VISCA......... 28
123. Price v. Innodata Philippines, Inc. ......................................................... 29
124. Agusan Del Norte Electric Cooperative vs. Cagampang ........................ 30
125. WILLIAM UY CONSTRUCTION CORP. and/or TERESITA UY and WILLIAM
UY vs. JORGE R. TRINIDAD ............................................................................ 31
126. Dacuital vs. L.M. Camus Engineering Corp., .......................................... 32
127. MILLENNIUM ERECTORS CORPORATION , vs. MAGALLANES ............... 33
128. EXODUS vs BISCOCHO ......................................................................... 34
129. LEYTE GEOTHERMAL vs PNOC EDC ..................................................... 36
130. ST. PAUL COLLEGE QUEZON CITY vs.. ANCHETA II ................................ 37
131. Lynvil Fishing Enterprises vs. Ariola, ..................................................... 39
132. D.M. Consunji, Inc. v Jamin ................................................................... 40


2

e. termination of employment
ii. effects: valid dismissal; illegal dismissal
98. Jenny M Agabon, et al. vs NLRC
FACTS: Private respondent Riviera Home Improvements, Inc. is engaged in the
business of selling and installing ornamental and construction materials. It
employed petitioners Virgilio Agabon and Jenny Agabon as gypsum board and
cornice installers on January 2, 1992
2
until February 23, 1999 when they were
dismissed for abandonment of work. Petitioners then filed a complaint for illegal
dismissal and payment of money claims
3
and on December 28, 1999, the Labor
Arbiter rendered a decision declaring the dismissals illegal and ordered private
respondent to pay the monetary claims. On appeal, the NLRC reversed the Labor
Arbiter because it found that the petitioners had abandoned their work, and were
not entitled to backwages and separation pay. The other money claims awarded by
the Labor Arbiter were also denied for lack of evidence.
5
Upon denial of their
motion for reconsideration, petitioners filed a petition for certiorari with the Court
of Appeals which ruled that the dismissal of the petitioners was not illegal because
they had abandoned their employment but ordered the payment of money claims.
Petitioners assert that they were dismissed because the private respondent refused
to give them assignments unless they agreed to work on a "pakyaw" basis when
they reported for duty on February 23, 1999. They did not agree on this
arrangement because it would mean losing benefits as Social Security System (SSS)
members. Petitioners also claim that private respondent did not comply with the
twin requirements of notice and hearing. Private respondent, on the other hand,
maintained that petitioners were not dismissed but had abandoned their work.
9

However, petitioners did not report for work because they had subcontracted to
perform installation work for another company. Petitioners also demanded for an
increase in their wage to P280.00 per day. When this was not granted, petitioners
stopped reporting for work and filed the illegal dismissal case.
10

1. What are the requirements to dismiss an employee?
To dismiss an employee, the law requires not only the existence of a just and valid
cause but also enjoins the employer to give the employee the opportunity to be
heard and to defend himself.
13
Article 282 of the Labor Code enumerates the just
causes for termination by the employer: (a) serious misconduct or willful
disobedience by the employee of the lawful orders of his employer or the latter's
representative in connection with the employee's work; (b) gross and habitual
neglect by the employee of his duties; (c) fraud or willful breach by the employee of
the trust reposed in him by his employer or his duly authorized representative; (d)
commission of a crime or offense by the employee against the person of his
employer or any immediate member of his family or his duly authorized
representative; and (e) other causes analogous to the foregoing.
2. What are the elements of Abandonment for it to be a ground for
dismissal?
Abandonment is the deliberate and unjustified refusal of an employee to resume
his employment.
14
It is a form of neglect of duty, hence, a just cause for termination
of employment by the employer.
15
For a valid finding of abandonment, these two
factors should be present: (1) the failure to report for work or absence without valid
or justifiable reason; and (2) a clear intention to sever employer-employee
relationship, with the second as the more determinative factor which is manifested
by overt acts from which it may be deduced that the employees has no more
intention to work. The intent to discontinue the employment must be shown by
clear proof that it was deliberate and unjustified.
16

3. Did petitioners abandon their work?
YES. In February 1999, petitioners were frequently absent having subcontracted for
an installation work for another company. Subcontracting for another company
clearly showed the intention to sever the employer-employee relationship with
private respondent. This was not the first time they did this. In January 1996, they
did not report for work because they were working for another company. Private
respondent at that time warned petitioners that they would be dismissed if this
happened again. Petitioners disregarded the warning and exhibited a clear
intention to sever their employer-employee relationship. The record of an
employee is a relevant consideration in determining the penalty that should be
meted out to him.
17

In Sandoval Shipyard v. Clave,
18
we held that an employee who deliberately
absented from work without leave or permission from his employer, for the
purpose of looking for a job elsewhere, is considered to have abandoned his job.
We should apply that rule with more reason here where petitioners were absent
because they were already working in another company.
The law imposes many obligations on the employer such as providing just
compensation to workers, observance of the procedural requirements of notice and
hearing in the termination of employment. On the other hand, the law also
recognizes the right of the employer to expect from its workers not only good
performance, adequate work and diligence, but also good conduct
19
and loyalty.
3

The employer may not be compelled to continue to employ such persons whose
continuance in the service will patently be inimical to his interests.
20

4. What are the procedures for dismissal that need to be observed?
The procedure for terminating an employee is found in Book VI, Rule I, Section 2(d)
of the Omnibus Rules Implementing the Labor Code:
Standards of due process: requirements of notice. In all cases of
termination of employment, the following standards of due process shall
be substantially observed:
I. For termination of employment based on just causes as defined in Article
282 of the Code:
(a) A written notice served on the employee specifying the ground or
grounds for termination, and giving to said employee reasonable
opportunity within which to explain his side;
(b) A hearing or conference during which the employee concerned, with
the assistance of counsel if the employee so desires, is given opportunity
to respond to the charge, present his evidence or rebut the evidence
presented against him; and
(c) A written notice of termination served on the employee indicating that
upon due consideration of all the circumstances, grounds have been
established to justify his termination.
In case of termination, the foregoing notices shall be served on the
employee's last known address.
Dismissals based on just causes contemplate acts or omissions attributable to the
employee while dismissals based on authorized causes involve grounds under the
Labor Code which allow the employer to terminate employees. A termination for an
authorized cause requires payment of separation pay. When the termination of
employment is declared illegal, reinstatement and full backwages are mandated
under Article 279. If reinstatement is no longer possible where the dismissal was
unjust, separation pay may be granted.
Procedurally, (1) if the dismissal is based on a just cause under Article 282, the
employer must give the employee two written notices and a hearing or opportunity
to be heard if requested by the employee before terminating the employment: a
notice specifying the grounds for which dismissal is sought a hearing or an
opportunity to be heard and after hearing or opportunity to be heard, a notice of
the decision to dismiss; and (2) if the dismissal is based on authorized causes under
Articles 283 and 284, the employer must give the employee and the Department of
Labor and Employment written notices 30 days prior to the effectivity of his
separation.
a. What if the dismissal was for a just or for an authorized caused and due
process was observed?
The dismissal is undoubtedly valid and the employer will not suffer any
liability.
b. What if the dismissal was without just or authorized cause but due
process was observed; or the dismissal is without just or authorized cause
and there was no due process?
In this case, Article 279 mandates that the employee is entitled to
reinstatement without loss of seniority rights and other privileges and full
backwages, inclusive of allowances, and other benefits or their monetary
equivalent computed from the time the compensation was not paid up to
the time of actual reinstatement.
c. What if the dismissal is for just and authorized cause but due process was
not observed?
The dismissal should be upheld. While the procedural infirmity cannot be
cured, it should not invalidate the dismissal. However, the employer
should be held liable for non-compliance with the procedural
requirements of due process.
5. Was the dismissal in the instant case valid despite lack of due process?
YES. The dismissal should be upheld because it was established that the petitioners
abandoned their jobs to work for another company. Private respondent, however,
did not follow the notice requirements and instead argued that sending notices to
the last known addresses would have been useless because they did not reside
there anymore. Unfortunately for the private respondent, this is not a valid excuse
because the law mandates the twin notice requirements to the employee's last
known address.
21
Thus, it should be held liable for non-compliance with the
procedural requirements of due process.
4

6. Is the private respondent liable for petitioners holiday pay, service
incentive leave pay and 13
th
month pay without deductions?
As a general rule, one who pleads payment has the burden of proving it. Even
where the employee must allege non-payment, the general rule is that the burden
rests on the employer to prove payment, rather than on the employee to prove
non-payment. The reason for the rule is that the pertinent personnel files, payrolls,
records, remittances and other similar documents which will show that overtime,
differentials, service incentive leave and other claims of workers have been paid
are not in the possession of the worker but in the custody and absolute control of
the employer.
41

In the case at bar, if private respondent indeed paid petitioners' holiday pay and
service incentive leave pay, it could have easily presented documentary proofs of
such monetary benefits to disprove the claims of the petitioners. But it did not,
except with respect to the 13th month pay wherein it presented cash vouchers
showing payments of the benefit in the years disputed.
42
Allegations by private
respondent that it does not operate during holidays and that it allows its employees
10 days leave with pay, other than being self-serving, do not constitute proof of
payment. Consequently, it failed to discharge the onus probandi thereby making it
liable for such claims to the petitioners.
99. JAKA FOOD P]ROCESSING CORP vs PACOT
FACTS: Respondents Darwin Pacot, Robert Parohinog, David Bisnar, Marlon
Domingo, Rhoel Lescano and Jonathan Cagabcab were earlier hired by petitioner
JAKA Foods Processing Corporation (JAKA, for short) until the latter terminated
their employment on August 29, 1997 because the corporation was "in dire
financial straits". It is not disputed, however, that the termination was effected
without JAKA complying with the requirement under Article 283 of the Labor Code
regarding the service of a written notice upon the employees and the Department
of Labor and Employment at least one (1) month before the intended date of
termination. In time, respondents separately filed with the regional Arbitration
Branch of the National Labor Relations Commission (NLRC) complaints for illegal
dismissal, underpayment of wages and nonpayment of service incentive leave and
13th month pay against JAKA and its HRD Manager, Rosana Castelo. After due
proceedings, the Labor Arbiter rendered a decision
3
declaring the termination illegal
and ordering JAKA and its HRD Manager to reinstate respondents with full
backwages, and separation pay if reinstatement is not possible.
1. What are the legal implications of a situation where an employee is
dismissed for cause but such dismissal was effected without the
employers compliance with the notice requirement under the Labor
Code.
This, certainly, is not a case of first impression. In the very recent case of Agabon vs.
NLRC,
8
we had the opportunity to resolve a similar question. Therein, we found that
the employees committed a grave offense, i.e., abandonment, which is a form of a
neglect of duty which, in turn, is one of the just causes enumerated under Article
282 of the Labor Code. In said case, we upheld the validity of the dismissal despite
non-compliance with the notice requirement of the Labor Code. However, we
required the employer to pay the dismissed employees the amount of P30,000.00,
representing nominal damages for non-compliance with statutory due process,
thus:
"Where the dismissal is for a just cause, as in the instant case, the lack of statutory
due process should not nullify the dismissal, or render it illegal, or ineffectual.
However, the employer should indemnify the employee for the violation of his
statutory rights, as ruled in Reta vs. National Labor Relations Commission. The
indemnity to be imposed should be stiffer to discourage the abhorrent practice of
dismiss now, pay later, which we sought to deter in the Serrano ruling. The
sanction should be in the nature of indemnification or penalty and should depend
on the facts of each case, taking into special consideration the gravity of the due
process violation of the employer.
The violation of petitioners right to statutory due process by the private
respondent warrants the payment of indemnity in the form of nominal damages.
The amount of such damages is addressed to the sound discretion of the court,
taking into account the relevant circumstances.
2. What are the implications of a dismissal for just cause under Article 282,
on one hand, and a dismissal for authorized cause under Article 283, on
the other.
A dismissal for just cause under Article 282 implies that the employee concerned
has committed, or is guilty of, some violation against the employer, i.e. the
employee has committed some serious misconduct, is guilty of some fraud against
the employer, or, as in Agabon, he has neglected his duties. Thus, it can be said that
the employee himself initiated the dismissal process.
On another breath, a dismissal for an authorized cause under Article 283 does not
necessarily imply delinquency or culpability on the part of the employee. Instead,
5

the dismissal process is initiated by the employers exercise of his management
prerogative, i.e. when the employer opts to install labor saving devices, when he
decides to cease business operations or when, as in this case, he undertakes to
implement a retrenchment program.
The clear-cut distinction between a dismissal for just cause under Article 282 and a
dismissal for authorized cause under Article 283 is further reinforced by the fact
that in the first, payment of separation pay, as a rule, is not required, while in the
second, the law requires payment of separation pay.
9

For these reasons, there ought to be a difference in treatment when the ground for
dismissal is one of the just causes under Article 282, and when based on one of the
authorized causes under Article 283.
Accordingly, it is wise to hold that: (1) if the dismissal is based on a just cause under
Article 282 but the employer failed to comply with the notice requirement, the
sanction to be imposed upon him should be tempered because the dismissal
process was, in effect, initiated by an act imputable to the employee; and (2) if the
dismissal is based on an authorized cause under Article 283 but the employer failed
to comply with the notice requirement, the sanction should be stiffer because the
dismissal process was initiated by the employers exercise of his management
prerogative.
3. In the case at bar, how should the failure to observed due process be
dealt with?
The records before us reveal that, indeed, JAKA was suffering from serious business
losses at the time it terminated respondents employment.
It is, therefore, established that there was ground for respondents dismissal, i.e.,
retrenchment, which is one of the authorized causes enumerated under Article 283
of the Labor Code. Likewise, it is established that JAKA failed to comply with the
notice requirement under the same Article. Considering the factual circumstances
in the instant case and the above ratiocination, we, therefore, deem it proper to fix
the indemnity at P50,000.00.
We likewise find the Court of Appeals to have been in error when it ordered JAKA to
pay respondents separation pay equivalent to one (1) month salary for every year
of service. This is because in Reahs Corporation vs. NLRC,
11
we made the following
declaration:
"The rule, therefore, is that in all cases of business closure or cessation of operation
or undertaking of the employer, the affected employee is entitled to separation
pay. This is consistent with the state policy of treating labor as a primary social
economic force, affording full protection to its rights as well as its welfare. The
exception is when the closure of business or cessation of operations is due to
serious business losses or financial reverses; duly proved, in which case, the right
of affected employees to separation pay is lost for obvious reasons. xxx".
(Emphasis supplied)
PERSONAL END NOTES:
In Agabon vs NLRC, the indemnity imposed by the court was P30,000.00 because
the employees were dismissed for just cause under Article 282. However, in this
case where the employees were dismissed for an authorized cause under Article
283, a higher indemnity is imposed in the amount of P50,000.00. This is because in
282, dismissal was initiated by the imputable act of the employee, while in 283, the
dismissal was initiated by the employer in the exercise of its management
prerogative which requires a stiffer sanction.
iv. preventive suspension; constructive dismissal; suspension of business
operation.
100. ALU-TUCP v. NLRC
FACTS:
Petitioners, as employees of private respondent National Steel Corporation
(NSC), filed separate complaints for unfair labor practice, regularization and
monetary benefits with the NLRC, Sub-Regional Arbitration Branch XII, Iligan City.
The complaints were consolidated and after hearing, the Labor Arbiter declared
petitioners regular project employees who shall continue their employment as
such for as long as such *project+ activity exists, but entitled to the salary of
a regular employee pursuant to the provisions in the collective bargaining
agreement. It also ordered payment of salary differentials.
The NLRC in its questioned resolutions modified the Labor Arbiters
decision. It affirmed the Labor Arbiters holding that petitioners were project
employees since they were hired to perform work in a specific undertaking the
Five Years Expansion Program, the completion of which had been determined at the
time of their engagement and which operation was not directly related to the
business of steel manufacturing. The NLRC, however, set aside the award to
petitioners of the same benefits enjoyed by regular employees for lack of legal and
factual basis. The law on the matter is Article 280 of the Labor Code, where the
petitioners argue that they are regular employees of NSC because: (i) their jobs
6

are necessary, desirable and work-related to private respondents main business,
steel-making; and (ii) they have rendered service for six (6) or more years to
private respondent NSC.
ISSUE:
Are petitioners are considered permanent employees?
HELD:
NO.
Petition for Certiorari dismissed for lack of merit. NLRC Resolutions
affirmed. Petitioners are not considered permanent employees. However,
contrary to petitioners apprehensions, the designation of named employees as
project employees and their assignment to a specific project are effected and
implemented in good faith, and not merely as a means of evading otherwise
applicable requirements of labor laws. On the claim that petitioners service to NSC
of more than six (6) years should qualify them as regular employees, the Supreme
Court believed this claim is without legal basis. The simple fact that the
employment of petitioners as project employees had gone beyond one (1) year,
does not detract from, or legally dissolve, their status as project employees. The
second paragraph of Article 280 of the Labor Code, quoted above, providing that an
employee who has served for at least one (1) year, shall be considered a regular
employee, relates to casual employees, not to project employees.
It is evidently important to become clear about the meaning and scope of
the term "project" in the present context. The "project" for the carrying out of
which "project employees" are hired would ordinarily have some relationship to the
usual business of the employer. Exceptionally, the "project" undertaking might not
have an ordinary or normal relationship to the usual business of the employer. In
this latter case, the determination of the scope and parameters of the "project"
becomes fairly easy. It is unusual (but still conceivable) for a company to undertake
a project which has absolutely no relationship to the usual business of the
company; thus, for instance, it would be an unusual steel-making company which
would undertake the breeding and production of fish or the cultivation of
vegetables. From the viewpoint, however, of the legal characterization problem
here presented to the Court, there should be no difficulty in designating the
employees who are retained or hired for the purpose of undertaking fish culture or
the production of vegetables as "project employees," as distinguished from
ordinary or "regular employees," so long as the duration and scope of the project
were determined or specified at the time of engagement of the "project
employees."
For, as is evident from the provisions of Article 280 of the Labor Code,
quoted earlier, the principal test for determining whether particular employees are
properly characterized as "project employees" as distinguished from "regular
employees," is whether or not the "project employees" were assigned to carry out a
"specific project or undertaking," the duration (and scope) of which were specified
at the time the employees were engaged for that project.
In the realm of business and industry, we note that "project" could refer to
one or the other of at least two (2) distinguishable types of activities. Firstly, a
project could refer to a particular job or undertaking that is within the regular or
usual business of the employer company, but which is distinct and separate, and
identifiable as such, from the other undertakings of the company. Such job or
undertaking begins and ends at determined or determinable times. The typical
example of this first type of project is a particular construction job or project of a
construction company. A construction company ordinarily carries out two or more
discrete identifiable construction projects: e.g., a twenty-five- storey hotel in
Makati; a residential condominium building in Baguio City; and a domestic air
terminal in Iloilo City. Employees who are hired for the carrying out of one of these
separate projects, the scope and duration of which has been determined and made
known to the employees at the time of employment, are properly treated as
"project employees," and their services may be lawfully terminated at completion
of the project.
The term "project" could also refer to, secondly, a particular job or
undertaking that is not within the regular business of the corporation. Such a job or
undertaking must also be identifiably separate and distinct from the ordinary or
regular business operations of the employer. The job or undertaking also begins and
ends at determined or determinable times. The case at bar presents what appears -
to our mind as a typical example of this kind of "project."
101. Cosmos Bottling Corp., vs NLRC, 255
SCRA 358 [1996]
Facts: Gil C. Castro was employed by Cosmos Bottling Corporation for a specific
period. Having satisfactorily served the company for two (2) terms, Castro was
recommended for reemployment with the companys Maintenance Team for the
Davao Project, he was re-hired and assigned to the Maintenance Division of the
Davao Project tasked to install the private respondents annex plant machines in its
Davao plant. Castros employment was terminated due to the completion of the
special project. Cosmos Bottling Corporation in valid exercise of its management
prerogative terminated the services of some 228 regular employees by reason of
retrenchment. For obvious reasons, Castro was not among the list of those regular
employees whose services were terminated by reason of retrenchment or those
7

who voluntarily resigned. Castro filed a complaint for illegal dismissal against
Cosmos Bottling Corporation with the Labor Arbiter contending that being a regular
employee, he could not be dismissed without a just and valid cause. The company
alleged that Castro was a mere project employee whose employment was co-
terminous with the project for which he was hired.

Issue: Whether or not private respondent Gil C. Castro is a regular employee or was
a mere project employee of petitioner Cosmos Bottling Corporation.

Ruling: Private respondent being a project employee, or to use the correct term,
seasonal employee, considering that his employment was limited to the installation
and dismantling of petitioners annex plant machines after which there was no
more work to do, his employment legally ended upon completion of the project.
That being so the termination of his employment cannot and should not constitute
an illegal dismissal. Neither should it constitute retrenchment as private
respondent was a seasonal employee whose services were already terminated on
May 21, 1990 prior to the termination of the other regular employees of Cosmos by
reason of retrenchment.

Article 280. Regular and Casual Employment. - The provisions of written
agreement to the contrary notwithstanding and regardless of the oral
agreement of the parties, an employment shall be deemed to be regular
where the employee has been engaged to perform activities which are
usually necessary or desirable in the usual business or trade of the
employer, except where the employment has been fixed for a specific
project or undertaking the completion or termination of which has been
determined at the time of the engagement of the employee or where the
work or services to be performed is seasonal in nature and the employment
is for the duration of the season.

An employment shall be deemed to be casual if it is not covered by the
preceding paragraph: Provided, That, any employee who has rendered at
least one year of service whether such service is continuous or broken, shall
be considered a regular employee with respect to the activity in which he is
employed and his employment shall continue while such actually exists.

The first paragraph provides that regardless of any written or oral agreement to the
contrary, an employee is deemed regular where he is engaged in necessary or
desirable activities in the usual trade or business of the employer.
A project employee, on the other hand, has been defined to be one whose
employment has been fixed for a specific project or undertaking, the completion or
termination of which has been determined at the time of the engagement of the
employee or where the work or service to be performed is seasonal in nature and
the employment is for the duration of the season.
The second paragraph of the provision defines casual employees as those who do
not fall under the definition of the first paragraph.

However, with respect to the first two kinds of employees, the principal test for
determining whether an employee is a project employee or a regular employee is
whether or not the project employee was assigned to carry out a specific project
or undertaking, the duration and scope of which were specified at the time the
employee was engaged for that period.

Petitioner Cosmos Bottling Corporation is a duly organized corporation engaged in
the manufacture, production, bottling, sale and distribution of beverage. In the
course of its business, it undertakes distinct identifiable projects as it did in the
instant case when it formed special teams assigned to install and dismantle its
annex plant machines in various plants all over the country. These projects are
distinct and separate, and are identifiable as such, from its usual business of
bottling beverage. Their duration and scope are made known prior to their
undertaking and their specified goal and purpose are fulfilled once the projects are
completed. When private respondent was initially hired for a period of one month
and re-hired for another five months, and then subsequently re-hired for another
five months, he was assigned to the petitioners Maintenance Division tasked with
the installation and dismantling of its annex plant machines. Evidently, these
projects or undertakings, the duration and scope of which had been determined
and made known to private respondent at the time of his employment, can
properly be treated as projects within the meaning of the first kind. Considered
as such, the services rendered by private respondent hired therein for the duration
of the projects may lawfully be terminated at the end or completion of the same.
Certainly, the lengthy gaps between his employments, together with the fact that
his services were contracted for specific undertakings, convincingly show that the
services of private respondent were terminated upon completion of a particular
project and were sought only when another one was undertaken. Moreover, the
mere fact that a project employee has worked on the specific project for more than
one (1) year, does not necessary change his status as project employee and convert
it to regular or permanent employment. For it is obvious that the second paragraph
of Article 280 of the Labor Code, quoted above, providing that an employee who
has served for at least one (1) year, shall be considered a regular employee, relates
only to casual employees, not to project employees
102. Purefoods Corporation vs. NLRC, et. al
G.R. No. 122653, December 12, 1997

8

FACTS:
The private respondents (numbering 906) were hired by petitioner Pure Foods
Corporation to work for a fixed period of five months at its tuna cannery plant in
Tambler, General Santos City. After the expiration of their respective contracts of
employment in June and July 1991, their services were terminated. They forthwith
executed a Release and Quitclaim stating that they had no claim whatsoever
against the petitioner. On December 1992, Private respondents filed before the
NLRC a complaint for illegal dismissal against the petitioner and its plant manager,
Marciano Aganon.

The Labor Arbiter dismissed the complaint on the ground that the private
respondents were mere contractual workers, and not regular employees; hence,
they could not avail of the law on security of tenure. The private respondents
appealed from the decision to the NLRC which affirmed the Labor Arbiter's
decision. On private respondents motion for reconsideration, the NLRC rendered
another decision on 30 January 1995 vacating and setting aside its earlier decision
and held that the private respondents and their co-complainants were regular
employees. It declared that the contract of employment for five months was a
clandestine scheme employed by *the petitioner+ to stifle *private respondents+
right to security of tenure and should therefore be struck down and disregarded
for being contrary to law, public policy, and morals. Hence, their dismissal on
account of the expiration of their respective contracts was illegal.

Petitioners motion for reconsideration was denied; hence, this appeal.
Petitioners submission before the Court: the private respondents are now
estopped from questioning their separation from petitioners employ in view of
their express conformity with the five-month duration of their employment
contracts. In the instant case, the private respondents were employed for a period
of five months only. In any event, private respondents' prayer for reinstatement is
well within the purview of the Release and Quitclaim they had executed wherein
they unconditionally released the petitioner from any and all other claims which
might have arisen from their past employment with the petitioner.

ISSUE: Whether or not the 5-month period specified in private respondents
employment contract is invalid and is therefore violative of their constitutional right
to security of tenure.

Ruling:
The five-month period specified in private respondents employment contract is
invalid. In
the leading case of Brent School, Inc. v. Zamora, although the Court has upheld the
legality of fixed-term employment, the Court also held that where from the
circumstances it is apparent that the periods have been imposed to preclude
acquisition of tenurial security by the employee, they should be struck down or
disregarded as contrary to public policy and morals.

Brent also laid down the criteria under which term employment cannot
be said to be in circumvention of the law on security of tenure: 1) The fixed period
of employment was knowingly and voluntarily agreed upon by the parties
without any force, duress, or improper pressure being brought to bear upon the
employee and absent any other circumstances vitiating his consent; or 2) It
satisfactorily appears that the employer and the employee dealt with each other
on more or less equal terms with no moral dominance exercised by the former or
the latter.

None of these criteria had been met in the present case. It could not be
supposed that private respondents and all other so-called casual workers of [the
petitioner] KNOWINGLY and VOLUNTARILY agreed to the 5-month employment
contract.
The petitioner does not deny or rebut private respondents' averments (1) that
the main bulk of its workforce consisted of its so-called casual employees; (2) that
as of July 1991, casual workers numbered 1,835; and regular employees, 263;
(3) that the company hired casual every month for the duration of five months,
after which their services were terminated and they were replaced by other
casual employees on the same five-month duration; and (4) that these casual
employees were actually doing work that were necessary and desirable in
petitioners usual business.
This scheme of the petitioner was apparently designed to prevent the
private respondents and the other casual employees from attaining the status
of a regular employee. It was a clear circumvention of the employees right to
security of tenure and to other benefits like minimum wage, cost-of-living
allowance, sick leave, holiday pay, and 13th month pay. Indeed, the petitioner
succeeded in evading the application of labor laws. Also, it saved itself from the
trouble or burden of establishing a just cause for terminating employees by the
simple expedient of refusing to renew the employment contracts.
The five-month period specified in private respondents employment contracts
having been imposed precisely to circumvent the constitutional guarantee on
security of tenure should, therefore, be struck down or disregarded as contrary to
public policy or morals. To uphold the contractual arrangement between the
9

petitioner and the private respondents would, in effect, permit the former to avoid
hiring permanent or regular employees by simply hiring them on a temporary or
casual basis, thereby violating the employees security of tenure in their jobs.
The NLRC was correct in finding that the private respondents were regular
employees and that they were illegally dismissed from their jobs. Under Article 279
of the Labor Code and the recent jurisprudence, the legal consequence of illegal
dismissal is reinstatement without loss of seniority rights and other privileges,
with full back wages computed from the time of dismissal up to the time of actual
reinstatement, without deducting the earnings derived elsewhere pending the
resolution of the case.
However, since reinstatement is no longer possible because the petitioner's
tuna cannery plant had, admittedly, been closed in November 1994, the proper
award is separation pay equivalent to one month pay or one-half month pay for
every year of service, whichever is higher, to be computed from the
commencement of their employment up to the closure of the tuna cannery
plant. The amount of back wages must be computed from the time the private
respondents were dismissed until the time petitioner's cannery plant ceased
operation.

Decision: WHEREFORE, for lack of merit, the instant petition is DISMISSED and the
challenged decision of 30 January 1995 of the National Labor Relations Commission
in NLRC CA No. M-001323-93 is hereby AFFIRMED subject to the above modification
on the computation of the separation pay and back wages.
103. PHILIPPINE FRUIT & VEGETABLE INDUSTRIES, INC. vs. NLRC
GR No. 122122
July 20, 1999

FACTS:
Private respondent Philippine Fruit and Vegetable Workers Union-Tupas Local
Chapter, for and in behalf of 127 of its members, filed a complaint for unfair labor
practice and/or illegal dismissal with damages against petitioner Philippine Fruit &
Vegetable Industries, Inc (PFVII). They alleged that the dismissals were due to
complainants' involvement in union activities and were without just cause.

Labor Arbiter rendered judgment PFVII was indeed guilty of Illegal Dismissal.

On appeal, NLRC set aside the Labor Arbiter's decision and remanded the said case
to the Arbitration Branch for further proceedings.

Arbitration Branch rendered a decision finding PFVII liable for illegal dismissal.

On appeal, NLRC affirmed the Arbitration Branchs decision but modified the
awards of attorneys fees.

PFVII filed a motion for reconsideration which was denied by NLRC.

Thus, this special civil action for certiorari.

One of PFVII's contentions is that the complainants are seasonal workers. According
to them, its operation starts only in February with the processing of tomatoes into
tomato paste and ceases by the end of the same month when the supply is
consumed. It then resumes operations at the end of April or early May, depending
on the availability of supply with the processing of mangoes into purees and ceases
operation in June. The severance of complainants' employment from petitioner
corporation was a necessary consequence of the nature of seasonal employment.


ISSUE:
Were the complainants seasonal workers, thus, they cannot invoke any tenurial
benefit?

RULING:
By the very nature of things in a business enterprise like PFVII, the services of the
complainants are, indeed, more than six (6) months a year. The company did not
confine itself just to the processing of tomatoes and mangoes. It also processed
guyabano, calamansi, papaya, pineapple, etc. Besides, they have the office of
administrative functions, cleaning and upkeeping of machines and other duties and
tasks to keep up a big food processing corporation. Considering, therefore, that
under Article 280 of the Labor Code "the provisions of written agreement to the
contrary notwithstanding and considering further that the tasks which
complainants performed were usually necessary and desirable in the employers
usual business or trade, Supreme Court held that complainants are regular seasonal
employees, thus, entitled to security of tenure. The Labor Code provides an
employment shall be deemed to be regular where the employee has been engaged
to perform activities that are usually necessary or desirable in the usual business or
trade of the employers, except where the employment has been fixed for a specific
project. An employment shall be deemed to be casual if it is not covered by the
preceding paragraph; provided, that, any employee who has rendered at least one
year of service whether such service is continuous or broken, shall be considered a
regular employee with respect to the activity in which he is employed and his
employment shall continue while such actually exists.
10


In the case at bar, the work of complainants as seeders, operators, sorters, slicers,
janitors, drivers, truck helpers, mechanics and office personnel is without doubt
necessary in the usual business of a food processing company like petitioner PFVII.

It should be noted that complainants' employment has not been fixed for a specific
project or undertaking the completion or termination of which has been
determined at the time of their appointment or hiring. Neither is their employment
seasonal in nature. While it may be true that some phases of petitioner company's
processing operations is dependent on the supply of fruits for a particular season,
the other equally important aspects of its business, such as manufacturing and
marketing are not seasonal. The fact is that large-scale food processing companies
such as PFVII continue to operate and do business throughout the year even if the
availability of fruits and vegetables is seasonal.

PETITION DENIED.

COMPLAINANTS ARE REGULAR EMPLOYEES BY VIRTUE OF THE FACT THAT THEY
PERFORMED FUNCTIONS WHICH ARE NECESSARY AND DESIRABLE IN THE USUAL
BUSINESS OF PFVII.

104. PHILIPS SEMICONDUCTORS (PHILS.), INC. V. FADRIQUELA
FACTS:
The petitioner Philips Semiconductors (Phils.), Inc. is a domestic corporation
engaged in the production and assembly of semiconductors such as power devices,
RF modules, CATV modules, RF and metal transistors and glass diods.
Aside from contractual employees, the petitioner employed 1,029 regular workers.
The employees were subjected to periodic performance appraisal based on output,
quality, attendance and work attitude. One was required to obtain a performance
rating of at least 3.0 for the period covered by the performance appraisal to
maintain good standing as an employee.

Respondent Eloisa Fadriquela executed a Contract of Employment with the
petitioner in which she was hired as a production operator. Her initial contract was
for a period of three months but was extended for two months when she garnered
a performance rating of 3.15. Her contract was again renewed for two months
when she received a performance rating of 3.8. After the expiration of her third
contract, it was extended anew, for three months.

After garnering a performance rating of 3.4, the respondents contract was
extended for another three months. She, however, incurred five absences in the
month of April, three absences in the month of May and four absences in the
month of June. Line supervisor Shirley F. Velayo asked the respondent why she
incurred the said absences, but the latter failed to explain her side. The respondent
was warned that if she offered no valid justification for her absences, Velayo would
have no other recourse but to recommend the non-renewal of her contract. The
respondent still failed to respond, as a consequence of which her performance
rating declined to 2.8. Velayo recommended to the petitioner that the respondents
employment be terminated due to habitual absenteeism, in accordance with the
Company Rules and Regulations. Thus, the respondents contract of employment
was no longer renewed.
The respondent filed a complaint before the National Capital Region Arbitration
Branch of the NLRC for illegal dismissal against the petitioner.
ISSUE & RULING:
Is respondent a regular employee of petitioner company, thus entitled to security of
tenure?
Yes. Article 280 of the Labor Code of the Philippines was emplaced in our statute
books to prevent the circumvention by unscrupulous employers of the employees
right to be secure in his tenure by indiscriminately and completely ruling out all
written and oral agreements inconsistent with the concept of regular employment
defined therein. The language of the law manifests the intent to protect the tenurial
interest of the worker who may be denied the rights and benefits due a regular
employee because of lopsided agreements with the economically powerful
employer who can maneuver to keep an employee on a casual or temporary status
for as long as it is convenient to it. In tandem with Article 281 of the Labor Code,
Article 280 was designed to put an end to the pernicious practice of making
permanent casuals of our lowly employees by the simple expedient of extending to
them temporary or probationary appointments, ad infinitum.
The two kinds of regular employees under the law are (1) those engaged to perform
activities which are necessary or desirable in the usual business or trade of the
employer; and (2) those casual employees who have rendered at least one year of
service, whether continuous or broken, with respect to the activities in which they
are employed. The primary standard to determine a regular employment is the
reasonable connection between the particular activity performed by the employee
in relation to the business or trade of the employer. The test is whether the former
is usually necessary or desirable in the usual business or trade of the employer. If
11

the employee has been performing the job for at least one year, even if the
performance is not continuous or merely intermittent, the law deems the repeated
and continuing need for its performance as sufficient evidence of the necessity, if
not indispensability of that activity to the business of the employer. Hence, the
employment is also considered regular, but only with respect to such activity and
while such activity exists. The law does not provide the qualification that the
employee must first be issued a regular appointment or must be declared as such
before he can acquire a regular employee status.
In this case, the respondent was employed by the petitioner on May 8, 1992 as
production operator. She was assigned to wirebuilding at the transistor division.
There is no dispute that the work of the respondent was necessary or desirable in
the business or trade of the petitioner. She remained under the employ of the
petitioner without any interruption since May 8, 1992 to June 4, 1993 or for one (1)
year and twenty-eight (28) days. The original contract of employment had been
extended or renewed for four times, to the same position, with the same chores.
Such a continuing need for the services of the respondent is sufficient evidence of
the necessity and indispensability of her services to the petitioners business. By
operation of law, then, the respondent had attained the regular status of her
employment with the petitioner, and is thus entitled to security of tenure as
provided for in Article 279 of the Labor Code which reads:
Art. 279. Security of Tenure. In cases of regular employment, the employer shall
not terminate the services of an employee except for a just cause or when
authorized by this Title. An employee who is unjustly dismissed from work shall be
entitled to reinstatement without loss of seniority rights and other privileges and to
his full backwages, inclusive of allowances, and to his other benefits or their
monetary equivalent computed from the time his compensation was withheld from
him up to the time of his actual reinstatement.
Under Section 3, Article XVI of the Constitution, it is the policy of the State to assure
the workers of security of tenure and free them from the bondage of uncertainty of
tenure woven by some employers into their contracts of employment. The
guarantee is an act of social justice. When a person has no property, his job may
possibly be his only possession or means of livelihood and those of his dependents.
When a person loses his job, his dependents suffer as well. The worker should
therefor be protected and insulated against any arbitrary deprivation of his job.
We reject the petitioners general and catch-all submission that its policy for a
specific and limited period on an "as the need arises" basis is not prohibited by law
or abhorred by the Constitution; and that there is nothing essentially contradictory
between a definite period of employment and the nature of the employees duties.
In the Romares v. NLRC case, we cited the criteria under which "term employment"
cannot be said to be in circumvention of the law on security of tenure, namely:
1) The fixed period of employment was knowingly and voluntarily agreed upon by
the parties without any force, duress, or improper pressure being brought to bear
upon the employee and absent any other circumstances vitiating his consent; or
2) It satisfactorily appears that the employer and the employee dealt with each
other on more or less equal terms with no moral dominance exercised by the
former or the latter.
None of these criteria has been met in this case.
105. Alcira vs NLRC
Facts:
The petitioner, Radin Alcira, was hired by the respondent Middleby Philippines
Corporation as engineering support services supervisor under probationary status
for 6 months. Afterwards, the service of the petitioner was terminated by
the respondent on the ground that the latter was not satisfied on the performance
of the former. As a result, the petitioner filed a complaint foe illegal dismissal in the
National Labor Relations Commission (NLRC) against the respondent.

Petitioner contended that his termination in the service tantamount to
illegal dismissal since he attained the status of a regular employee as of the time
of dismissal. He presented the appointment paper showing that he was hired on
May 20, 1996, consequently, his dismissal on November 20, 1996 was illegal
because at that time, he was already a regular employee since the 6-month
probationary period ended on November 16, 1996.

The respondent, on the other hand, asserted that during the petitioners
probationary period, he showed poor performance on his assigned tasks, was late
couple of times and violated the companys rule. Thus, the petitioner was
terminated and his application to become a regular employment was disapproved.
The respondent also insisted that the removal of the petitioner from office was
within the probationary period.

The Labor Arbiter dismissed the complaint on the ground that the dismissal of the
petitioner was done before his regularization because the 6- month probationary
period, counting from May 20, 1996 shall end on November 20, 1996. The NLRC
affirmed the decision of the Labor Arbiter. The Court of Appeals affirmed the
decision of NLRC. Hence, the present recourse.
12

Issue:
Whether the petitioner was already a regular employee in respondents company
at the time of his dismissal from the service.
Ruling:
The Supreme Court ruled in the negative. The status of the petitioner at the time
of his termination was still probationary. His dismissal on November 20, 1996 was
within the 6- month probationary period. Article 13 of the Civil Code provides that
when the law speaks of years, months, and days and nights, it shall be understood
that years are of 365 days, months of 30 days, days of 24 hours and nights are from
sunset to sunrise. Since, one month is composed of 30 days, then, 6 months shall be
understood to be composed of 180 days. And the computation of the 6- month
period is reckoned from the date of appointment up to the same calendar date
of the 6th month following. Since, the number of days of a particular month is
irrelevant, petitioner was still a probationary employee at the time of his dismissal.
Wherefore, the petition is dismissed.
106. Mitsubishi v. crysler
G.R. No. 148738
June 29, 2004

FACTS: Private respondent Nelson Paras first worked with Mitsubishi Philippines as
a shuttle bus driver on March 19, 1976. He resigned on June 16, 1982 because he
went to Saudi Arabia and worked there as a diesel mechanic and heavy machine
operator from 1982 to 1993. Upon his return, Mitsubishi Philippines re-hired him as
a welder-fabricator at a tooling shop from November 1, 1994 to March 3, 1995.

On May 1996, Paras was re-hired again, this time as a probationary manufacturing
trainee at the Plant Engineering Maintenance Department. He had an orientation
on May 15, 1996 and afterwhich, with respect to the companys rules and
guidelines, started reporting for work on May 27, 1996.

Paras was evaluated by his immediate supervisors after six months of working. The
supervisors rating Paras performance were Lito R. Lacambacal and Wilfredo J.
Lopez, as part of the MMPCs company policies. Upon this evaluation, Paras
garnered an average rating.

Later, respondent Paras was informed by his supervisor, Lacambacal, that he
received an average performance rating but it is a rate which would still qualify him
to be regularized. But as part of the company protocols, the Division Managers
namely A.C. Velando, H.T. Victoria and Dante Ong reviewed the performance
evaluation made on Paras. Despite the recommendations of the supervisors, they
unanimously agreed that the performance was unsatisfactory. As a consequence,
Paras was not considered for regularization.

Paras received a Notice of Termination on November 26, 1996 which was dated
November 25, 1996. This letters intent is to formally relieve him off of his services
and position effective the date since he failed to meet the companys standards.
ISSUE: Whether or not respondent Paras termination was legal or not.
HELD: The Court holds that a company employer may indeed hire an employee on a
probationary basis in order to determine his fitness to perform work. The Court
stresses the existence of the statements under Article 281 of the Labor Code which
specifies that the employer must inform the employee of the standards they were
to meet in order to be granted regularization and that such probationary period
shall not exceed six (6) months from the date the employee started working, unless
specified in the apprenticeship agreement.

Respondent Paras was employed on a probationary basis and was apprised of the
standards upon which his regularization would be based during the orientation. His
first day to report for work was on May 27, 1996. As per the company's policy, the
probationary period was from three (3) months to a maximum of six (6) months.
Applying Article 13 of the Civil Code, the probationary period of six (6) months
consists of one hundred eighty (180) days. The Court conforms with paragraph one,
Article 13 of the Civil Code providing that the months which are not designated by
their names shall be understood as consisting of thirty (30) days each. This case, the
Labor Code pertains to 180 days. Also, as clearly provided for in the last paragraph
of Article 13, it is said that in computing a period, the first day shall be excluded and
the last day included. Thus, the one hundred eighty (180) days commenced on May
27, 1996, and ended on November 23, 1996. The termination letter dated
November 25, 1996 was served on respondent Paras only at 3:00 a.m. of November
26, 1996. The Court held that by that time, he was actually already a regular
employee of the petitioner under Article 281 of the Labor Code. His position as a
regularized employee is thus secured until further notice.
107. PANGILINAN et. al. v. GENERAL MILLING CORP.
Facts
Respondent is a domestic corporation engaged in the production and sale of
livestock and poultry and distributor of dressed chicken to various restaurants and
establishments nationwide. It employs hundreds of employees, some on a regular
basis and others on a casual basis, as emergency workers.
13

Petitioners were employed by the respondent on different dates as emergency
workers at its poultry plant in Cainta, Rizal, under separate temporary/casual
contracts of employment for a period of five months. Upon the expiration of their
respective contracts, their services were terminated. They later filed separate
complaints for illegal dismissal and non-payment of holiday pay, 13
th
month pay,
night-shift differential and service incentive leave pay against the respondent
before the Arbitration Branch of the NLRC.
Petitioners alleged that their work as chicken dressers was necessary and desirable
in the usual business of the respondent, and added that although they worked from
10:00 p.m. to 6:00 a.m., they were not paid night-shift differential. They stressed
that based on the nature of their work, they were regular employees of the
respondent; hence, could not be dismissed from their employment unless for just
cause and after due notice. They further argued that the respondent could not rely
on the nomenclature of their employment as temporary or casual.
Labor Arbiter Balitaan ruled in favor of the petitioners, declaring that they were
regular employees and that they were allegedly illegally dismissed. The NLRC
rendered a decision reversing that of the Labor Arbiter, holding that petitioners,
who were temporary or contractual employees of the respondent, were legally
terminated upon the expiration of their respective contracts. Petitioners MR was
denied by the NLRC. The CA rendered a decision affirming with modification the
decision of the NLRC. The CA ruled that where the duties of the employee consist of
activities usually necessary or desirable in the usual business of the employer, it
does not necessarily follow that the parties are forbidden from agreeing on a period
of time for the performance of such activities. The petitioners filed an MR of the
said decision, which was denied.
Issue
What type of employees are petitioners? (It follows that if they are regular
employees, then they were illegally dismissed.)
Ruling
Petitioners were employees with a fixed period, and, as such, were not regular
employees.
Article 280 of the Labor Code comprehends three kinds of employees: (a) regular
employees or those whose work is necessary or desirable to the usual business of
the employer; (b) project employees or those whose employment has been fixed
for a specific project or undertaking the completion or termination of which has
been determined at the time of the engagement of the employee or where the
work or services to be performed is seasonal in nature and the employment is for
the duration of the season; and, (c) casual employees or those who are neither
regular nor project employees.
A regular employee is one who is engaged to perform activities which are necessary
and desirable in the usual business or trade of the employer as against those which
are undertaken for a specific project or are seasonal. There are two separate
instances whereby it can be determined that an employment is regular: (1) if the
particular activity performed by the employee is necessary or desirable in the usual
business or trade of the employer; and, (2) if the employee has been performing
the job for at least a year.
Article 280 of the Labor Code does not proscribe or prohibit an employment
contract with a fixed period. It does not necessarily follow that where the duties of
the employee consist of activities usually necessary or desirable in the usual
business of the employer, the parties are forbidden from agreeing on a period of
time for the performance of such activities. There is thus nothing essentially
contradictory between a definite period of employment and the nature of the
employees duties.
108. Ravago v. Esso Eastern Marine, Ltd.
G.R. No. 158324, March 14, 2005.
Facts:
Esso is engaged in maritime commerce. Ravago was hired as a seaman on
board various Esso vessels. He was employed under a total of 34 separate and
unconnected contracts, each for a fixed period, by three different companies,
namely, ETI, EEM and EIS.
After completing his latest contract, Ravago was shot by a stray bullet
which landed on his left leg. Because of such incident, he was declared unfit to work
once again as a seaman. Instead of rehiring Ravago, EIS paid him his Career
Employment Incentive Plan.
Ravago filed a complaint for illegal dismissal.
Essos contention: Ravago was not illegally dismissed because he was a
contractual employee and was hired under 34 separate contracts by different
companies.
14

Ravagos contentions: 1) He was not merely a contractual employee
because Esso regularly and continuously rehired him for 23 years, and 2) He was a
regular employee because he was engaged to perform activities which were usually
necessary or desirable in the usual trade or business of the employer.
Issue:
Was Ravago a regular employee of Esso?
Ruling:
NO. The Court has consistently ruled that seafarers are contractual, not
regular employees. Seamen and overseas contract workers are not covered by the
term regular employment as defined under Article 280 of the Labor Code. It should
have no application to instances where a fixed period of employment was agreed
upon knowingly and voluntarily by the parties, without any force, duress or
improper pressure being brought to bear upon the employee and absent any other
circumstances vitiating his consent, or where it satisfactorily appears that the
employer and employee dealt with each (Brent School, Inc. v. Zamora)
The Standard Employment Contract governing the Employment of All
Filipino Seamen on Board Ocean-Going Vessels of the POEA, particularly in Part I,
Sec. C, specifically provides that the contract of seamen shall be for a fixed period.
And in no case should the contract of seamen be longer than 12 months. Ravagos
employment have automatically ceased upon the expiration of his contract of
enlistment.
It is an accepted maritime industry practice that employment of seafarers
is for a fixed period only. Constrained by the nature of their employment which is
quite peculiar and unique in itself, it is for the mutual interest of both the seafarer
and the employer why the employment status must be contractual only or for a
certain period of time. Seafarers spend most of their time at sea and
understandably, they cannot stay for a long and an indefinite period of time at sea.
Limited access to shore society during the employment will have an adverse impact
on the seafarer. The national, cultural and lingual diversity among the crew during
the contract of enlistment is a reality that necessitates the limitation of its period
109. Hacienda Bino/Hortencia Starke vs. Cuenca
G.R. No. 150478, April 15, 2005

Hacienda Bino, represented by owner and operator Hortencia L. Starke, is a 236-
hectare sugar plantation in Kabankalan City. The 76 individual respondents were
part of its workforce, consisting of 220 workers, performing various tasks in the
planting and harvesting of sugarcane.

In 1996, during the off-milling season, Starke issued an Order of Notice to the
employees, which stated: Please bear in mind that all those who signed in favor of
CARP are expressing their desire to get out of employment on their own volition.
Wherefore, beginning today, July 18, only those who did not sign for CARP will be
given employment by Hda. Bino.

Respondents filed a complaint for illegal dismissal, wage differentials, 13th month
pay, holiday pay and premium pay for holiday, service incentive leave pay, and
moral and exemplary damages with the NLRC. In their Joint Sworn Statement, they
alleged that they were dismissed because they applied as beneficiaries under the
Comprehensive Agrarian Reform Program (CARP) over the land owned by Starke.

The Labor Arbiter found that Starkes notice was tantamount to a termination of
the respondents services, thus guilty of illegal dismissal. Petitioner was ordered to
pay backwages and wage differentials. The NLRC affirmed the decision of the Labor
Arbiter, with additional payment of holiday pay. The CA deleted the award for
holiday pay and premium pay for holidays.

ISSUE: WON the CA violated the doctrine of stare decisis laid down by the
Supreme Court as to the status of the sugar workers.

RULING: NO.

Starke contends that the CA violated the doctrine of stare decisis in not applying the
ruling in Mercado vs. NLRC, which held that sugar workers are seasonal employees,
whose employment legally ends upon completion of the project or the season.
Thus, the employment of respondents, being sugar workers, may be terminated at
the end of the season and such termination cannot be considered an illegal
dismissal.

Under the doctrine of stare decisis, when a court has laid down a principle of law as
applicable to a certain state of facts, it will adhere to that principle and apply it to
all future cases in which the facts are substantially the same. Where the facts are
essentially different, however, stare decisis does not apply, for a perfectly sound
principle as applied to one set of facts might be entirely inappropriate when a
factual variance is introduced. The CA correctly found that the facts involved in this
case are different from the Mercado case; therefore, the ruling in that case cannot
be applied to the case at bar.

15

Respondents in the instant case are REGULAR EMPLOYEES. Although in the
Mercado case, the Supreme Court held the sugar workers were not regular but
seasonal workers, nevertheless, the same does not operate to abandon the settled
doctrine that sugar workers are considered regular and permanent farm workers of
a sugar plantation owner. The disparity in facts between the Mercado case and the
instant case is best exemplified by the fact that the former decision ruled on the
status of employment of farm laborers, who, as found by the labor arbiter, work
only for a definite period for a farm worker, after which they offer their services to
other farm owners, considering the area in question being comparatively small,
comprising of seventeen and a half (17) hectares of land, such that the planting of
rice and sugar cane thereon could not possibly entail a whole year operation. The
herein case presents a different factual condition as the enormity of the size of the
sugar hacienda of petitioner, with an area of two hundred thirty-six (236) hectares,
simply do not allow for private respondents to render work only for a definite
period.

The primary standard for determining regular employment is the reasonable
connection between the particular activity performed by the employee in relation
to the usual trade or business of the employer. There is no doubt that the
respondents were performing work necessary and desirable in the usual trade or
business of an employer. Hence, they can properly be classified as regular
employees. For respondents to be excluded from those classified as regular
employees, it is not enough that they perform work or services that are seasonal in
nature. They must have been employed only for the duration of one season. While
the records sufficiently show that the respondents work in the hacienda was
seasonal in nature, there was, however, no proof that they were hired for the
duration of one season only. In fact, the payrolls, submitted in evidence by the
petitioners, show that they availed the services of the respondents since 1991.
Absent any proof to the contrary, the general rule of regular employment should,
therefore, stand.
110. PHILIPPINE GLOBAL COMMUNICATIONS, INC. vs. RICARDO DE VERA
G.R. No. 157214. June 7, 2005.*
Facts:
Petitioner PhilCom, is a corporation engaged in the business of communication
services and allied activities, while respondent Ricardo De Vera is a physician by
profession whom petitioner enlisted to attend to the medical needs of its
employees.
Respondent offered his services to petitioner and gave a proposal of his plan of
works required in industrial medicine. This proposal was formalized by petitioner
and respondent in a document denominated as Retainership contract. Said contract
was renewed yearly and it went on from 1981 to 1996. However, on dec. 1996,
petitioner informed respondent the termination of its retainership contract. This
led respondent to file an action for illegal dismissal before the NLRC alleging that
that he had been actually employed by petitioner as its company physician since
1981 and was dismissed without due process.
The LA dismissed the complaint and held that respondent was an independent
contractor and he was not dismissed but rather his contract ended when said
contract was not renewed. The NLRC, however, reversed the decision of the LA and
held that respondent is petitioners regular employee and, among others, directed
the latter to reinstate respondent to his former position.
Issues:
1. Whether or not an employer-employee relationship exists between
petitioner and respondent?
2. Whether or not respondent can be considered as regular employee of
petitioner company?
Ruling:
1. Applying the four-fold test to this case, there existed NO employer-
employee relationship between petitioner and respondent, because of the
following facts:
a. respondent himself sets the parameters of what his duties in
offering his services to petitioner.
b. respondent PHILCOM did not have control over the schedule of
the complainant as it [is] the complainant who is proposing his
own schedule and asking to be paid for the same. This is proof
that the complainant understood that his relationship with the
respondent PHILCOM was a retained physician and not as an
employee. If he were an employee he could not negotiate as to
his hours of work.
c. Respondent was never included in petitioners payroll; was never
deducted any contribution for remittance to the Social Security
System (SSS); and was in fact subjected by petitioner to the ten
(10%) percent withholding tax for his professional fee, in
accordance with the NIRC, matters which are simply inconsistent
16

with an employer-employee relationship. In the precise words of
the labor arbiter:
Clearly, the elements of an employer-employee relationship are
wanting in this case. Remarkably absent from the parties
arrangement is the element of control, whereby the employer has
reserved the right to control the employee not only as to the result of
the work done but also as to the means and methods by which the
same is to be accomplished.
Here, petitioner had no control over the means and methods by which
respondent went about performing his work at the company
premises. He could even embark in the private practice of his
profession, not to mention the fact that respondents work hours and
the additional compensation therefor were negotiated upon by the
parties. In fine, the parties themselves practically agreed on every
terms and conditions of respondents engagement, which thereby
negates the element of control in their relationship.
2. NO. The appellate courts premise that regular employees are those who
perform activities which are desirable and necessary for the business of
the employer is not determinative in this case. For, we take it that any
agreement may provide that one party shall render services for and in
behalf of another, no matter how necessary for the latters business, even
without being hired as an employee. This set-up is precisely true in the
case of an independent contractorship as well as in an agency
agreement. Indeed, Article 280 of the Labor Code, quoted by the appellate
court, is not the yardstick for determining the existence of an employment
relationship. As it is, the provision merely distinguishes between two (2)
kinds of employees, i.e., regular and casual. It does not apply where, as
here, the very existence of an employment relationship is in dispute.
Buttressing his contention that he is a regular employee of
petitioner, respondent invokes Article 157 of the Labor Code, and
argues that he satisfies all the requirements thereunder.
However, Petitioners business of telecommunications is not hazardous in
nature. As such, what applies here is the last paragraph of Article 157
which, to stress, provides that the employer may engage the services of a
physician and dentist on retained basis, subject to such regulations as
the Secretary of Labor may prescribe. The successive retainership
agreements of the parties definitely hue to the very statutory provision
relied upon by respondent.
Nowhere does the law provide that the physician or dentist so engaged
thereby becomes a regular employee. The very phrase that they may be
engaged on retained basis, revolts against the idea that this engagement
gives rise to an employer-employee relationship.
With the recognition of the fact that petitioner consistently engaged the
services of respondent on a retainer basis, as shown by their various
retainership contracts, so can petitioner put an end, with or without
cause, to their retainership agreement as therein provided.
111. Integrated Contractor and Plumbing Works, Inc. vs. NLRC
G.R. No. 152427. August 9, 2005
Facts:

Petitioner Integrated Contractor and Plumbing Works Inc. is a plumbing contractor.
Its business depends on the number and frequency of the projects it is able to
contract with its clients. Respondent Glen Solon works for petitioner. On February
23, 1998, while private respondent was about to log out from work, he was
informed by the warehouseman that the main office had instructed them to tell him
it was his last day of work as he had been terminated. When private respondent
went to the petitioner's office to verify his status, he found out that indeed, he had
been terminated. He filed a complaint alleging that he was illegally dismissed
without just cause and without due process.

The Labor Arbiter ruled that private respondent was a regular employee and could
only be removed for cause. Petitioner was ordered to reinstate private respondent
to his former position with full backwages from the time his salary was withheld
until his actual reinstatement, and pay him service incentive leave pay, and 13th
month pay for three years.

Petitioner further filed a motion for reconsideration which was denied. It filed an
appeal before the CA but it was subsequently dismissed for lack of merit.

Issue:

1. Whether the respondent is a project employee of the petitioner or a
regular employee.
2. Was respondent validly dismissed?
17


HELD

1. No. He was considered as a regular employee.
As held in Tomas Lao Construction v. NLRC, the principal test in determining
whether an employee is a "project employee" or "regular employee," is
whether he is assigned to carry out a "specific project or undertaking," the
duration (and scope) of which are specified at the time the employee is
engaged in the project. "Project" refers to a particular job or undertaking that is
within the regular or usual business of the employer, but which is distinct and
separate and identifiable from the undertakings of the company. Such job or
undertaking begins and ends at determined or determinable times.
A review of private respondent's work assignments patently showed
he belonged to a work pool tapped from where workers are and assigned
whenever their services were needed. In a work pool, the workers do not
receive salaries and are free to seek other employment during temporary
breaks in the business. They are like regular seasonal workers insofar as the
effect of temporary cessation of work is concerned. This arrangement is
beneficial to both the employer and employee for it prevents the unjust
situation of "coddling labor at the expense of capital" and at the same time
enables the workers to attain the status of regular employees. 15 Nonetheless,
the pattern of re-hiring and the recurring need for his services are sufficient
evidence of the necessity and indispensability of such services to petitioner's
business or trade.
In Maraguinot, Jr. v. NLRC, it was held that once a project or work pool
employee has been: (1) continuously, as opposed to intermittently, re-hired by
the same employer for the same tasks or nature of tasks; and (2) these tasks
are vital, necessary and indispensable to the usual business or trade of the
employer, then the employee must be deemed a regular employee.
The test to determine whether employment is regular or not is the
reasonable connection between the particular activity performed by the
employee in relation to the usual business or trade of the employer. Also, if the
employee has been performing the job for at least one year, even if the
performance is not continuous or merely intermittent, the law deems the
repeated and continuing need for its performance as sufficient evidence of the
necessity, if not indispensability of that activity to the business. Thus, we held
that where the employment of project employees is extended long after the
supposed project has been finished, the employees are removed from the
scope of project employees and are considered regular employees.
While length of time may not be the controlling test for project
employment, it is vital in determining if the employee was hired for a specific
undertaking or tasked to perform functions vital, necessary and indispensable
to the usual business or trade of the employer. Here, private respondent had
been a project employee several times over. His employment ceased to be
coterminous with specific projects when he was repeatedly re-hired due to the
demands of petitioner's business. 20 Where from the circumstances it is
apparent that periods have been imposed to preclude the acquisition of
tenurial security by the employee, they should be struck down as contrary to
public policy, morals, good customs or public order.
Further, Policy Instructions No. 20 requires employers to submit a
report of an employee's termination to the nearest public employment office
every time his employment was terminated due to a completion of a project.
The failure of the employer to file termination reports is an indication that the
employee is not a project employee. 22 Department Order No. 19 superseding
Policy Instructions No. 20 also expressly provides that the report of termination
is one of the indications of project employment. 23 In the case at bar, there was
only one list of terminated workers submitted to the Department of Labor and
Employment. 24 If private respondent was a project employee, petitioner
should have submitted a termination report for every completion of a project
to which the former was assigned.
2. No, he was illegally dismissed. As a regular worker, private respondent is
entitled to security of tenure under Article 279 of the Labor Code and can only
be removed for cause. The Court found no valid cause attending to private
respondents dismissal and found also that his dismissal was without due
process.
Additionally, Article 277(b) of the Labor Code provides that
... Subject to the constitutional right of workers to security of tenure and their
right to be protected against dismissal except for a just and authorized cause
and without prejudice to the requirement of notice under Article 283 of this
Code, the employer shall furnish the worker whose employment is sought to be
terminated a written notice containing a statement of the causes for
termination and shall afford the latter ample opportunity to be heard and to
defend himself with the assistance of his representative if he so desires in
accordance with company rules and regulations promulgated pursuant to
guidelines set by the Department of Labor and Employment
The failure of the petitioner to comply with these procedural guidelines
renders its dismissal of private respondent, illegal. An illegally dismissed
employee is entitled to reinstatement with full backwages, inclusive of
allowances, and to his other benefits computed from the time his
compensation was withheld from him up to the time of his actual
reinstatement, pursuant to Article 279 of the Labor Code.
18

112. LOLITA R. LACUESTA vs. ATENEO DE MANILA UNIVERSITY
G.R. No. 152777. December 9, 2005.*
FACTS: Respondent Ateneo de Manila University (Ateneo) hired, on a contractual
basis, petitioner Lolita R. Lacuesta as a part-time lecturer in its English Department
for two semesters from 1988-1990. On 1990, the petitioner was first appointed as
full-time instructor on probation, in the same department effective June 1, 1990
until March 31, 1991. Thereafter, her contract as faculty on probation was renewed
for two years from April 1,1991 to March 31,1993. During these three years she was
on probation status. In a letter dated January 27, 1993, respondent Dr. Garcia, Dean
of Ateneos Graduate School and College of Arts and Sciences, notified petitioner
that her contract would no longer be renewed because she did not integrate well
with the English Department. Petitioner then appealed to the President of the
Ateneo at the time, Fr. Joaquin Bernas, S.J.
In a letter, Fr. Bernas explained to petitioner that she was not being
terminated, but her contract would simply expire. Fr. Bernas offered petitioner the
job as book editor in the University Press under terms comparable to that of a
faculty member.
On March 26, 1993, petitioner applied for clearance to collect her final salary
as instructor. Petitioner also signed a Quitclaim, Discharge and Release on April 16,
1993.
Petitioner worked as editor in the University Press from April 1, 1993 to March
31, 1994 including an extension of two months after her contract expired. Upon
expiry of her contract, petitioner applied for clearance to collect her final salary as
editor. Later, she agreed to extend her contract from June 16, 1994 to October 31,
1994. Petitioner decided not to have her contract renewed due to a severe back
problem. She did not report back to work, but she submitted her clearance on
February 20, 1995.
On December 23, 1996, petitioner filed a complaint for illegal dismissal with
prayer for reinstatement, back wages, and moral and exemplary damages.
LA DECISION: petitioner may not be terminated by mere lapse of the probationary
period but only for just cause or failure to meet the employers standards. The
quitclaim, discharge and release executed by petitioner was not a bar to filing a
complaint for illegal dismissal. Thus, he ordered reinstatement with payment of full
back wages.
NLRC DECISION: Reversed the Labor Arbiters decision
CA DECISION: dismissed the petition and affirmed the NLRC decision.
ISSUE: whether or not respondent was illegally dismissed?
RULING: NO. The Manual of Regulations for Private Schools, and not the Labor
Code, determines whether or not a faculty member in an educational institution
has attained regular or permanent status. In University of Santo Tomas v. National
Labor Relations Commission the Court en banc said that under Policy Instructions
No. 11 issued by the Department of Labor and Employment, the probationary
employment of professors, instructors and teachers shall be subject to the
standards established by the Department of Education and Culture. Said standards
are embodied in paragraph 75 (now Section 93) of the Manual of Regulations for
Private Schools.
Section 93 of the 1992 Manual of Regulations for Private Schools provides
that full-time teachers who have satisfactorily completed their probationary
period shall be considered regular or permanent. Moreover, for those teaching in
the tertiary level, the probationary period shall not be more than six consecutive
regular semesters of satisfactory service. The requisites to acquire permanent
employment, or security of tenure, are (1) the teacher is a full-time teacher; (2)
the teacher must have rendered three consecutive years of service; and (3) such
service must have been satisfactory.
As previously held, a part-time teacher cannot acquire permanent status.
Only when one has served as a full-time teacher can he acquire permanent or
regular status. The petitioner was a part-time lecturer before she was appointed
as a full-time instructor on probation. As a part-time lecturer, her employment as
such had ended when her contract expired. Thus, the three semesters she served
as part-time lecturer could not be credited to her in computing the number of
years she has served to qualify her for permanent status.
Petitioner posits that after completing the three-year probation with an
above-average performance, she already acquired permanent status. On this point,
we are unable to agree with petitioner.
Completing the probation period does not automatically qualify her to
become a permanent employee of the university. Petitioner could only qualify to
become a permanent employee upon fulfilling the reasonable standards for
permanent employment as faculty member. Consistent with academic freedom
and constitutional autonomy, an institution of higher learning has the prerogative
to provide standards for its teachers and determine whether these standards have
been met. At the end of the probation period, the decision to re-hire an employee
on probation, belongs to the university as the employer alone.
19

We reiterate, however, that probationary employees enjoy security of tenure,
but only within the period of probation. Likewise, an employee on probation can
only be dismissed for just cause or when he fails to qualify as a regular employee in
accordance with the reasonable standards made known by the employer at the
time of his hiring. Upon expiration of their contract of employment, academic
personnel on probation cannot automatically claim security of tenure and compel
their employers to renew their employment contracts. In the instant case,
petitioner, did not attain permanent status and was not illegally dismissed. As
found by the NLRC, her contract merely expired.
Lastly, we find that petitioner had already signed a valid quitclaim, discharge
and release which bars the present action. This Court has held that not all
quitclaims are per se invalid or against public policy, except (1) where there is clear
proof that the waiver was wangled from an unsuspecting or gullible person, or (2)
where the terms of settlement are unconscionable on their face. In this case, there
is no showing that petitioner was coerced into signing the quitclaim. In her sworn
quitclaim, she freely declared that she received to her full satisfaction all that is due
her by reason of her employment and that she was voluntarily releasing respondent
Ateneo from all claims in relation to her employment. Nothing on the face of her
quitclaim has been shown as unconscionable.
113. POSEIDON FISHING VS NLRC
G.R. No. 168052. February 20, 2006.
FACTS: Petitioner Poseidon Fishing is a fishing company engaged in the deep-sea
fishing industry. One of its boat crew was private respondent Jimmy S. Estoquia.
Petitioner Terry de Jesus is the manager of petitioner company. Private respondent
was employed by Poseidon Fishing in January 1988 as Chief Mate. After five years,
he was promoted to Boat Captain. In 1999, petitioners, without reason, demoted
respondent from Boat Captain to Radio Operator of petitioner Poseidon. As a Radio
Operator, he monitored the daily activities in their office and recorded in the duty
logbook the names of the callers and time of their calls. On 3 July 2000, private
respondent failed to record a 7:25 a.m. call in one of the logbooks. However, he
was able to record the same in the other logbook. Consequently, when he
reviewed the two logbooks, he noticed that he was not able to record the said call
in one of the logbooks so he immediately recorded the 7:25 a.m. call after the 7:30
a.m. entry. Around 9:00 oclock in the morning of 4 July 2000, petitioner Terry de
Jesus detected the error in the entry in the logbook. Subsequently, she asked
private respondent to prepare an incident report to explain the reason for the said
oversight.
That same day, petitioner Poseidons secretary, summoned private respondent to
get his separation pay amounting to (P55,000.00). However, he refused to accept
the amount as he believed that he did nothing illegal to warrant his immediate
discharge from work. Rising to the occasion, private respondent filed a complaint
for illegal dismissal on 11 July 2000 with the Labor Arbiter, alleging nonpayment of
wages with prayer for back wages, damages, attorneys fees, and other monetary
benefits.
Conversely, petitioners Poseidon and Terry de Jesus strongly asserted that private
respondent was a contractual or a casual employee whose services could be
terminated at the end of the contract even without a just or authorized cause in
view of Article 280 of the Labor Code. Asserting their right to terminate the contract
with private respondent per the Kasunduan with him, petitioners pointed to the
provision thereof stating that he was being employed only on a por viaje basis
and that his employment would be terminated at the end of the trip for which he
was being hired. Quoting Brent School Inc. v. Zamora, petitioners are hamstrung on
their reasoning that under the Civil Code, fixed-term employment contracts are not
limited, as they are under the present Labor Code, to those that by their nature are
seasonal or for specific projects with pre-determined dates of completion as they
also include those to which the parties by free choice have assigned a specific date
of termination. Hence, persons may enter into such contracts as long as they are
capacitated to act, petitioners bemoan.

1. Is the Kasunduan between private respondent and petitioner valid so
that the former cannot be considered as a regular employee?
NO. In the case under consideration, the agreement has such an objective - to
frustrate the security of tenure of private respondent- and fittingly, must be
nullified. In this case, petitioners intent to evade the application of Article 280 of
the Labor Code is unmistakable. In a span of 12 years, private respondent worked
for petitioner company first as a Chief Mate, then Boat Captain, and later as Radio
Operator. His job was directly related to the deep-sea fishing business of petitioner
Poseidon. His work was, therefore, necessary and important to the business of his
employer. Such being the scenario involved, private respondent is considered a
regular employee of petitioner under Article 280 of the Labor Code, the law in
point, which provides:
Art. 280. Regular and Casual Employment. The
provisions of written agreement to the contrary notwithstanding
and regardless of the oral agreement of the parties, an
employment shall be deemed to be regular where the employee
has been engaged to perform activities which are usually
20

necessary or desirable in the usual business or trade of the
employer, except where the employment has been fixed for a
specific project or undertaking the completion or termination of
which has been determined at the time of the engagement of the
employee or where the work or services to be performed is
seasonal in nature and the employment is for the duration of the
season.

An employment shall be deemed to be casual if it is not
covered by the preceding paragraph: Provided, That any
employee who has rendered at least one year of service, whether
such service is continuous or broken, shall be considered a regular
employee with respect to the activity in which he is employed and
his employment shall continue while such actually exists.
(Emphasis supplied.)


Moreover, unlike in the Brent case where the period of the contract was
fixed and clearly stated, note that in the case at bar, the terms of employment of
private respondent as provided in the Kasunduan was not only vague, it also failed
to provide an actual or specific date or period for the contract.

Furthermore, as petitioners themselves admitted in their petition before
this Court, private respondent was repeatedly hired as part of the boats crew and
he acted in various capacities onboard the vessel. In Integrated Contractor and
Plumbing Works, Inc. v. National Labor Relations Commission, we held that the test
to determine whether employment is regular or not is the reasonable connection
between the particular activity performed by the employee in relation to the usual
business or trade of the employer. And, if the employee has been performing the
job for at least one year, even if the performance is not continuous or merely
intermittent, the law deems the repeated and continuing need for its performance
as sufficient evidence of the necessity, if not indispensability of that activity to the
business.

Ostensibly, in the case at bar, at different times, private respondent
occupied the position of Chief Mate, Boat Captain, and Radio Operator. In
petitioners interpretation, however, this act of hiring and re-hiring actually
highlight private respondents contractual status saying that for every engagement,
a fresh contract was entered into by the parties at the outset as the conditions of
employment changed when the private respondent filled in a different position.
But to this Court, the act of hiring and re-hiring in various capacities is a mere
gambit employed by petitioner to thwart the tenurial protection of private
respondent. Such pattern of re-hiring and the recurring need for his services are
testament to the necessity and indispensability of such services to petitioners
business or trade.
114. Abesco Construction and Dev. Corp. v Ramirez
GR 141168 (2006)
Facts
Petitioner, ACDC, a construction company, hired respondents on different dates
from 1976-1992 either as laborers, road roller operators, painters or drivers.
Respondents filed complaints for illegal dismissal against ACDC and its General
Manager. Allegedly, ACDC dismissed respondents without valid grounds and
without due process. The complaint also included claims for non-payment of the
13th month pay, 5 days' serivce incentive leave pay, premium pay for holidays and
rest days, and moral and exemplary damages.
ACDC avers that respondents were merely project employees since their services
were necessary only when the company had projects. Being project employees, not
regular employees, their employment was coterminous with the project to be
undertaken; they were not entitled to separation pay.
LA: Respondents are regular employees because they belonged to the "work pool"
from which ACDC drew workers for assignment to different projects, at its
discretion. Respondents were hired and re-hired over a period of 18 years, hence,
they were deemed to be regular employees. Termination was without just cause.
NLRC: Affirmed LA ruling.
CA: Dismissed appeal. Petitioners raised new defenses for the first time, which is
not allowed. (Previous defense: respondents were project employees who were not
entitled to security of tenure. New defense: Respondents were not dismissed but
their employment was merely suspended.)
MR dismissed.
Issues
Are respondents regular or project employees?
Respondents were regular employees.
21

Is the length of service a controlling factor in determinig the nature of one's
employment?
No. Employees (like respondents) who work under different project employment
contracts for several years do not automatically become regular employees; they
can remain as project employees regardless of the number of years they
work.Length of service is not a controlling factor in determining the nature of ones
employment.
Are employees who are part of a "work pool" regular employees?
Not necessarily. Employees who are members of a work pool from which a
company (like petitioner corporation) draws workers for deployment to its different
projects do not become regular employees by reason of that fact alone. The Court
has enunciated in some cases that members of a work pool can either be project
employees or regular employees.
What is the test for determining whether employees are "project" or "regular
employees"?
The principal test for determining whether employees are project employees or
regular employees is whether they are assigned to carry out a specific project or
undertaking, the duration and scope of which are specified at the time they are
engaged for that project.[10] Such duration, as well as the particular work/service
to be performed, is defined in an employment agreement and is made clear to the
employees at the time of hiring.
In this case, petitioners did not have that kind of agreement with respondents.
Neither did they inform respondents of the nature of the latters work at the time
of hiring. Hence, for failure of petitioners to substantiate their claim that
respondents were project employees, we are constrained to declare them as
regular employees.
On illegal dismissal
Respondents were illegally dismissed because petitioners failed to adhere to the
two-notice rule which requires that workers to be dismissed must be furnished
with:
1. a notice informing them of the particular acts for which they are being
dismissed; and
2. a notice advising them of the decision to terminate the employment.
Respondents were never given such notices.
115. Cebu Metal v. Saliling, 501 SCRA 61
FACTS:
Respondent (Cebu Metal Corporation) is a corporation engaged in buying
and selling of scrap iron. In the Bacolod Branch, they have employees who are
undertaking pakiao work in the unloading of scrap iron for stockpiling.
Among those workers who presented for work in the unloading of scrap
iron in the area are the unemployed persons or trisicad drivers standing by in the
vicinity, most of whom are the herein complainants.
The company alleged that its Bacolod Branch is mainly a stockyard where
scrap metal delivered by its suppliers are stockpiled. The trucks used in the delivery
of scrap metal are owned and/or rented by the different suppliers of scrap metal.
Sometimes, the suppliers do not have any truck boys, and in these instances, the
respondent hires the services of people for the purpose of unloading the scrap
metal from these trucks. Usually, there is a leader for a particular group who is also
tasked to distribute the individual take of each member.
The complainants raised their protest saying that they never received any
other benefits from the company; 13th month pay, holiday pay, etc. So, when the
complainants demanded from the company for the increase of their salary, the
manager got irritated and told them to stop working. Hence, this petition.
ISSUE: WON said employees are regular employees.
NEGATIVE. A painstaking review of the decision of the NLRC will readily reveal that
the Commissions finding that respondent complainants were not regular
employees was the raison dtre for the subsequent turnaround of the state of
affairs. What the NLRC made use of to reverse the Labor Arbiters decision was
precisely the conclusion of the latter that respondent complainants were regular
employees of petitioner company. According to the Commission, such conclusion
was predicated merely on the consideration that respondent complainants were
performing activities necessary and desirable to the business or trade of their
employer. Based on the facts of the case and the evidence presented by the parties
to the case at bar, however, the NLRC arrived at a divergent conclusion, which we
fully agree in. We quote with approval its disquisition:
It is interesting to note that the Labor Arbiter had given credence and
probative value to the Petty Cash Vouchers submitted by the respondents. Thus he
said:
22

The petty cash vouchers (Annexes 1 to1-A-62, respondents position
paper) show that complainants are not paid on hourly or daily basis as they would
like this office to believe but on pakiao or task basis at P15.00 per metric ton.
There is no basis then for complainants to claim that they are underpaid since
there is no minimum wage in this type of work. Complainants earnings depend
upon their own diligence and speed in unloading and stockpiling scrap iron. More
importantly, it depends upon the availability of scrap iron to be unloaded and
stockpiled.
The above findings validate respondents position as to the nature of
complainants work. Their services are needed only when scrap metals are
delivered which occurs only one or twice a week or sometimes no delivery at all in
a given week. The irregular nature of work, stoppage of work and then work again
depending on the supply of scrap metal has not been denied by complainants. On
the contrary they even admitted the same in their Reply to respondents Appeal. x x
x. Indeed, it would be unjust to require respondent to maintain complainants in the
payroll even if there is no more work to be done. To do so would make
complainants privileged retainers who collect payment from their employer for
work not done. This is extremely unfair and amount to cuddling of labor at the
expense of management.
116. LIGANZA vs RBL SHIPYARD CORPORATION
G.R. No. 159862 , October 17, 2006

HERMONIAS L. LIGANZA, Petitioner vs. RBL SHIPYARD CORPORATION and ENGR.
BEN LIM, JR., Respondents.

FACTS:

After working as a carpenter for respondent since August 1991, petitioners
employment was terminated on 30 October 1999. This prompted petitioner to file a
complaint for illegal dismissal alleging that on said date he was verbally informed
that he was already terminated from employment and barred from entering the
premises. On the same occasion, he was told to look for another job. Thus, he
claimed that he was unceremoniously terminated from employment without any
valid or authorized cause. On the other hand, respondent insisted that petitioner
was a mere project employee who was terminated upon completion of the project
for which he was hired.

Petitioner claims he is a regular employee since he worked for respondent
continuously and without interruption from August 13, 1991 up to October 30,
1999 (8 yrs) and that his work as a carpenter was necessary and desirable to the
latters usual business of shipbuilding and repair. He asserts that when he was hired
by respondent in 1991, there was no employment contract fixing a definite period
or duration of his engagement, except for the contract covering the period
September 20, 1999 to March 19, 2000, respondent had been unable to show the
other project employment contracts ever since petitioner started working for the
company. Furthermore, respondent failed to file as many termination reports as
there are completed projects involving petitioner, he adds.

On the other hand, respondent insists that petitioner is a project employee as
evidenced by the project employment contracts it signed with him and employee
termination reports it submitted to the DOLE.

ISSUE: Whether or not petitioner is a project employee of respondent company.

HELD:

NO.

The SC held that a project employee is one whose "employment has been fixed for
a specific project or undertaking, the completion or termination of which has been
determined at the time of the engagement of the employee or where the work or
service to be performed is seasonal in nature and the employment is for the
duration of the season." Before an employee hired on a per project basis can be
dismissed, a report must be made to the nearest employment office of the
termination of the services of the workers every time it completed a project,
pursuant to Policy Instruction No. 20.

While the appropriate evidence to show that a person is a project employee is the
employment contract specifying the project and the duration of such project, the
existence of such contract is not always conclusive of the nature of ones
employment. In the instant case, respondent seeks to prove the status of
petitioners employment through four (4) employment contracts covering a period
of only two (2) years to declare petitioner as a project employee.

All that respondent submitted were four (4) contracts covering the periods
(1) July 29, 1997 to January 28, 1998,
(2) August 24, 1998 to February 25, 1999,
(3) 3 March 1999 to 2 September 1999, and
(4) 20 September 1999 to 19 March 2000,

as well as the employment termination reports for January 1998, August 1998,
February 1999 and October 1999. Respondent failed to present the contracts
purportedly covering petitioners employment from 1991 to July 1997, spanning six
23

(6) years of the total eight (8) years of his employment. To explain its failure in this
regard, respondent claims that the records and contracts covering said period were
destroyed by rains and flashfloods that hit the companys office. We are not
convinced.

To begin with, respondent has been unable to refute petitioners allegation that he
did not sign any contract when he started working for the company. The four
employment contracts are not sufficient to reach the conclusion that petitioner
was, and has been, a project employee earlier since 1991. The Court is not
satisfied with the explanation that the other employment contracts were
destroyed by floods and rains. Respondent could have used other evidence to
prove project employment, but it did not do so, seemingly content with the
convenient excuse of destroyed documents.

Even assuming that petitioner is a project employee, respondent failed to prove
that his termination was for a just and valid cause. While it is true that the
employment contract states that the contract ends upon a specific date, or upon
completion of the project, respondent failed to prove that the last project was
indeed completed so as to justify petitioners termination from employment.

In termination cases, the burden of proof rests on the employer to show that the
dismissal is for a just cause. Thus, employers who hire project employees are
mandated to state and, once its veracity is challenged, to prove the actual basis for
the latters dismissal. Respondent could have easily proved that the project or
phase for which petitioner was hired has already been completed. A certificate
from the owner of the vessel serviced by the company, pictures perhaps, of the
work accomplished, and other proof of completion could have been procured by
respondent. However, all that we have is respondents self-serving assertion that
the project has been completed.

This Court has held that an employment ceases to be co-terminous with specific
projects when the employee is continuously rehired due to the demands of
employers business and re-engaged for many more projects without
interruption. Once a project or work pool employee has been: (1) continuously, as
opposed to intermittently, rehired by the same employer for the same tasks or
nature of tasks; and (2) these tasks are vital, necessary and indispensable to the
usual business or trade of the employer, then the employee must be deemed a
regular employee, pursuant to Article 280 of the Labor Code and jurisprudence.

Surely, length of time is not the controlling test for project employment.
Nevertheless, it is vital in determining if the employee was hired for a specific
undertaking or tasked to perform functions vital, necessary and indispensable to
the usual business or trade of the employer.[ Here, respondent had been a project
employee several times over. His employment ceased to be coterminous with
specific projects when he was repeatedly re-hired due to the demands of
petitioners business. Where from the circumstances it is apparent that periods
have been imposed to preclude the acquisition of tenurial security by the
employee, they should be struck down as contrary to public policy, morals, good
customs or public order.

All considered, there are serious doubts in the evidence on record that petitioner is
a project employee, or that he was terminated for just cause. These doubts shall be
resolved in favor of petitioner, in line with the policy of the law to afford protection
to labor and construe doubts in favor of labor.

It is well-settled that the employer must affirmatively show rationally adequate
evidence that the dismissal was for a justifiable cause. When there is no showing of
a clear, valid and legal cause for the termination of employment, the law considers
the matter a case of illegal dismissal and the burden is on the employer to prove
that the termination was for a valid or authorized cause. For failure to prove
otherwise, the Court has no recourse but to grant the petition.
117. Fabela, et. al vs. San Miguel Corporation
G.R. no. 150658, February 9, 2007


FACTS:
Petitioners were hired by respondent San Miguel Corporation (SMC) as
Relief Salesmen for the Greater Manila Area (GMA) under separate but almost
similarly worded Contracts of Employment With Fixed Period. After having
entered into successive contracts of the same nature with SMC, the services of
petitioners were terminated after SMC no longer agreed to forge another contract
with them. SMC claimed that the hiring of route salesmen, like Fabela and others,
was not intended to be permanent.
SMC was undergoing a gradual transition from route system to a new system of
selling and delivering its products the Pre-Selling System in which the salesmen
under the earlier system would be replaced by Accounts Specialists which called for
upgraded qualifications.
While some of the qualified regular salesmen were readily upgraded to the
position of Accounts Specialist, SMC argued that it still had to sell its beer products
using the conventional routing system during the transition stage, thus giving rise to
the need for temporary employees. The route salesmen then existing were required
to undergo a training program to determine whether they possessed or could be
trained as Accounts Specialists. Hence, it claimed that the hiring of petitioners and
24

others for a fixed period was co-terminus with the completion of the transition
period and Training Program for all prospective Accounts Specialists.
Claiming that they were illegally dismissed, petitioners filed separate
complaints for illegal dismissal against respondents. The Labor Arbiter held that the
complainants, herein petitioners, were illegally dismissed. The Decision of the
Labor Arbiter was affirmed on appeal by the NLRC, by Resolution of April 28,
2000. Respondents Motion for Reconsideration was denied, hence, they filed a
Petition for Certiorari with the Court of Appeals before which they contended that
herein petitioners were validly hired for a fixed period which was not renewed,
hence, the termination of their services was valid. The Court of Appeals granted
respondents petition and accordingly reversed the decision of the Labor Arbiter
and of the NLRC. The appellate court accordingly dismissed petitioners
complaints. Their motion for reconsideration having been denied by the Court of
Appeals, petitioners filed the present petition.

ISSUE: Was there a valid fixed period employment?

Ruling:
No. Although Article 280 does not expressly recognize employment for a
fixed period, which is distinct from employment which has been fixed for a specific
project or undertaking, it has been clarified that employment for a fixed period is
not in itself illegal. Even if the duties of an employee consist of activities usually
necessary or desirable in the usual business of the employer, it does not necessarily
follow that the parties are forbidden from agreeing on a period of time for the
performance of such activities through a contract of employment for a fixed term.
SMCs contention that there are fixed periods stated in the contracts of
employment does not lie. The ruling in Brent instructs that a contract of
employment stipulating a fixed-term, even if clear as regards the existence of a
period, is invalid if it can be shown that the same was executed with the intention
of circumventing security of tenure, and should thus be ignored.
Indeed, substantial evidence exists in the present case showing that the
subject contracts were utilized to deprive petitioners of their security of tenure.
The contract of employment of petitioner Fabela, for instance, states that the
transition period from the Route System to the Pre-Selling System would be twelve
(12) months from April 4, 1995. It bears noting, however, that petitioner Fabela,
besides being hired again for another fixed period of four (4) months after the lapse
in April 1996 of the one-year contract, had already been working for respondent
SMC on a fixed-term basis as early as 1992, or one year before respondent SMC
even began its shift to the Pre-selling System in 1993.
A fixed-term employment is valid only under certain circumstances, such
as when the employee himself insists upon the period, or where the nature of the
engagement is such that, without being seasonal or for a specific project, a
definite date of termination is a sine qua non.
Respondents have not established that the engagement of petitioners
services, which is not in the nature of a project employment, required a definite
date of termination as a sine qua non. IN FINE, the finding of the Labor Arbiter and
the NLRC that the execution of the contracts was merely intended to circumvent
petitioners security of tenure merits this Courts concurrence.
118. SORIANO vs. NLRC and PLDT
GR No. 165594, April 23,, 2007

FACTS:
Petitioner Soriano and certain individuals: Benjamin, Gonzales, and Apostol were
employed by the respondent as Switchman Helpers in its Tondo Exchange Office
(TEO). After participating in several trainings and seminars, petitioner, Benjamin,
and Gonzales were promoted as Switchmen. Apostol, on the other hand, was
elevated to the position of Frameman. One of their duties as Switchmen and
Frameman was the manual operation and maintenance of the Electronic
Mechanical Device (EMD) of the TEO.

In November 1995, respondent PLDT implemented a company-wide redundancy
program. Due to the following reasons:
a) Technological changes where new technologies necessitate reduction in
workforce, e.g., conversion of electro-mechanical switches; outmoded
electronic switches to modern digital switches.

b) Position declared redundant due to collapsing/merging of functions
where the required number of personnel became less, i.e. rehoming of toll
centers or centralization of toll centers.

c) Non-replacement of function upon retirement of executive where
attached staffs with the executive are no longer needed Staff Assistant,
Secretary, Clerk.

d) Process Improvements and Automation of functions which render the
positions as redundant since the new process or Automation require less
personnel.

e) Functions or positions which are affected adversely by market forces,
thereby necessitating reduction of current workforce to match the reduction
of workload, i.e., Traffic due to decreasing number of handled calls.

Subsequently, the respondent PLDT gave separate letters to petitioner, Benjamin,
Gonzales, and Apostol informing them that their respective positions were deemed
25

redundant due to the above-cited reasons and that their services will be terminated
soon thereafter. They requested the respondent PLDT for transfer to some vacant
positions but their requests were denied since all positions were already filled up.
Hence, respondent PLDT dismissed the four from employment.
Thereafter, petitioner, Benjamin, Gonzales, and Apostol filed a joint
complaint for illegal dismissal against respondent PLDT.
The Labor Arbiter opined that respondent PLDT's redundancy program was
effected in good faith as the reduction of the latter's employees was brought about
by its adoption of the latest communication technology equipment which can be
operated by computers alone.
NLRC promulgated its Decision dismissing the appeal and affirming in toto
the decision of Labor Arbiter.
The CA dismissed the Petition and found no grave abuse of discretion on
the part of the NLRC in rendering its assailed Decision and Resolution.

ISSUE:
Were the four employees illegally dismissed?

RULING:
No. The Labor Arbiter, the NLRC, and the CA all found that substantial evidence
supports the absence of illegal dismissal in the present case. Article 283 of the
Labor Code provides that an employer may dismiss from work an employee by
reason of redundancy. The same provision also states the procedural requirements
for the validity of the dismissal. It is clear that the foregoing documentary evidence
constituted substantial evidence to support the findings of Labor Arbiter and the
NLRC that petitioner's employment was terminated by respondent PLDT due to a
valid or legal redundancy program since substantial evidence merely refers to that
amount of evidence which a reasonable mind might accept as adequate to support
a conclusion.

The records show that respondent PLDT had sufficiently established the existence
of redundancy in the position of Switchman. It is evident from the foregoing facts
that respondent PLDT's utilization of high technology equipment in its operation
such as computers and digital switches necessarily resulted in the reduction of the
demand for the services of a Switchman since computers and digital switches can
aptly perform the function of several Switchmen. Indubitably, the position of
Switchman has become redundant.

The fact that respondent PLDT hired contractual employees after implementing its
redundancy program does not necessarily negate the existence of redundancy. As
amply stated by the respondent PLDT, such hiring was intended solely for winding
up operations using the old system. Since the respondent PLDT determined that
petitioner's services are no longer necessary either as a Switchman or in any other
position, and such determination was made in good faith and in furtherance of its
business interest, the petitioner's contention that he should be the last switchman
to be laid-off by reason of his qualifications and outstanding work must fail.

119. Caseres vs. Universal Robina Sugar Milling Corporation (URSUMCO)
FACTS:
Universal Robina Sugar Milling Corporation (respondent) is a corporation engaged
in the cane sugar milling business. Pedy Caseres (petitioner Caseres) started
working for respondent in 1989, while Andito Pael (petitioner Pael) in 1993. At the
start of their respective employments, they were made to sign a Contract of
Employment for Specific Project or Undertaking. Petitioners contracts were
renewed from time to time, until May 1999 when they were informed that their
contracts will not be renewed anymore.
Petitioners filed a complaint for illegal dismissal, regularization, incentive
leave pay, 13th month pay, damages and attorneys fees.
In a Decision dated August 24, 1999, the Labor Arbiter (LA) dismissed the
complaint for not being substantiated with clear and convincing evidence.
The National Labor Relations Commission (NLRC) affirmed the LAs
dismissal, and the Court of Appeals (CA) dismissed the petition filed before it.
Hence, herein Petition for Review on Certiorari under Rule 45 of the Rules of Court.
ISSUES & RULING:
1) WHETHER OR NOT THE PETITIONERS ARE SEASONAL/PROJECT/TERM EMPLOYEES
NOT REGULAR EMPLOYEES OF RESPONDENTS.
The petition is without merit.
The rule is clear that a petition for review on certiorari under Rule 45 of
the Rules of Court should raise only questions of law, subject to certain exceptions.
Whether or not respondents were project employees or regular employees is a
question of fact.
The LA, the NLRC and the CA are one in ruling that petitioners were not
illegally dismissed as they were not regular, but contractual or project employees.
Consequently, the finding of the LA, the NLRC, and the CA that petitioners
were project employees binds this Court.
26

The Court finds no cogent reason to depart from their ruling.
Article 280 of the Labor Code provides:
ART. 280. Regular and Casual Employees.The provision of written
agreement to the contrary notwithstanding and regardless of the oral agreement of
the parties, an employment shall be deemed to be regular where the employee has
been engaged to perform activities which are usually necessary or desirable in the
usual business or trade of the employer, except where the employment has been
fixed for a specific project or undertaking the completion or termination of which
has been determined at the time of the engagement of the employee or where the
work or services to be performed is seasonal in nature and the employment is for
the duration of the season.
An employment shall be deemed to be casual if it is not covered by the
preceding paragraph: Provided, That, any employee who has rendered at least one
year of service, whether such service is continuous or broken, shall be considered a
regular employee with respect to the activity in which he is employed and his
employment shall continue while such actually exists.
The foregoing provision provides for three kinds of employees: (a) regular
employees or those who have been engaged to perform activities which are
usually necessary or desirable in the usual business or trade of the employer; (b)
project employees or those whose employment has been fixed for a specific
project or undertaking, the completion or termination of which has been
determined at the time of the engagement of the employee or where the work or
services to be performed is seasonal in nature and the employment is for the
duration of the season; and (c) casual employees or those who are neither regular
nor project employees.
The principal test for determining whether an employee is a project
employee or a regular employee is whether the employment has been fixed for a
specific project or undertaking, the completion or termination of which has been
determined at the time of the engagement of the employee. A project employee is
one whose employment has been fixed for a specific project or undertaking, the
completion or termination of which has been determined at the time of the
engagement of the employee or where the work or service to be performed is
seasonal in nature and the employment is for the duration of the season. A true
project employee should be assigned to a project which begins and ends at
determined or determinable times, and be informed thereof at the time of hiring.
The fact that petitioners were constantly re-hired does not ipso facto
establish that they became regular employees. Their respective contracts with
respondent show that there were intervals in their employment. In petitioner
Caseress case, while his employment lasted from August 1989 to May 1999, the
duration of his employment ranged from one day to several months at a time, and
such successive employments were not continuous. With regard to petitioner Pael,
his employment never lasted for more than a month at a time. These support the
conclusion that they were indeed project employees, and since their work
depended on the availability of such contracts or projects, necessarily the
employment of respondents work force was not permanent but co-terminous with
the projects to which they were assigned and from whose payrolls they were paid.
As ruled in Palomares v. National Labor Relations Commission, it would be
extremely burdensome for their employer to retain them as permanent employees
and pay them wages even if there were no projects to work on.
Moreover, even if petitioners were repeatedly and successively re-hired,
still it did not qualify them as regular employees, as length of service is not the
controlling determinant of the employment tenure of a project employee, but
whether the employment has been fixed for a specific project or undertaking, its
completion has been determined at the time of the engagement of the employee.
Further, the proviso in Article 280, stating that an employee who has rendered
service for at least one (1) year shall be considered a regular employee, pertains
to casual employees and not to project employees.
2) WHETHER OR NOT THE PETITIONERS WERE ILLEGALLY DISMISSED AND ARE
ENTITLED TO BACKWAGES AND OTHER MONETARY BENEFITS PRAYED FOR IN THE
COMPLAINT.
No. Petitioners cannot complain of illegal dismissal inasmuch as the
completion of the contract or phase thereof for which they have been engaged
automatically terminates their employment.
120. Pier 8 Arrastre and Stevedoring Services Inc. vs. Boclot
Facts:
Boclot was hired by PASSI to perform the functions of a stevedore. Later on, Boclot
filed Complaint with the Labor Arbiter claiming regularization; payment of service
incentive leave and 13th month pays; moral, exemplary and actual damages; and
attorneys fees.
He alleged that he was hired by PASSI in October 1999 and was issued company ID
No. 304, a PPA Pass and SSS documents. In fact, respondent contended that he
became a regular employee by April 2000, since it was his sixth continuous month
in service in PASSIs regular course of business. He argued on the basis of Articles
27

280 and 281 of the Labor Code. He maintains that under paragraph 2 of Article 280,
he should be deemed a regular employee having rendered at least one year of
service with the company.
Issue:
Whether or not he has attained regular status
Ruling:
Yes. SC took judicial notice that it is an industry practice in port services to hire
reliever stevedores in order to ensure smooth-flowing 24-hour stevedoring and
arrastre operations in the port area. No doubt, serving as a stevedore, respondent
performs tasks necessary or desirable to the usual business of petitioners. However,
it should be deemed part of the nature of his work that he can only work as a
stevedore in the absence of the employee regularly employed for the very same
function.
- Though usual and necessary, his employment is dependent on availability of
work
Moreover, respondent does not contest that he was well aware that he would only
be given work when there are absent or unavailable employees. Respondent also
does not allege, nor is there any showing, that he was disallowed or prevented from
offering his services to other cargo handlers in the other piers at the North Harbor
other than petitioners. As aforestated, the situation of respondent is akin to that of
a seasonal or project or term employee, albeit on a daily basis.
The Supreme Court still finds respondent to be a regular employee on the basis of
pertinent provisions under the CBA between PASSI and its Workers union, wherein
it was stated that it agrees to convert to regular status all incumbent probationary
or casual employees and workers in the Company who have served the Company
for an accumulated service term of employment of not less than six (6) months
from his original date of hiring.
-Under the CBA, he qualifies as a regular employee
Respondent assents that he is not a member of the union, as he was not recognized
by PASSI as its regular employee, but this Court notes that PASSI adopts a union-
shop agreement, culling from Article II of its CBA. Under a union-shop agreement,
although non members may be hired, an employee is required to become a union
member after a certain period, in order to retain employment. This requirement
applies to present and future employees. The same article of the CBA stipulates
that employment in PASSI cannot be obtained without prior membership in the
union.
Hence, applying the foregoing provisions of the CBA, respondent should be
considered a regular employee after six months of accumulated service. Having
rendered 228.5 days, or eight months of service to petitioners since 1999, then
respondent is entitled to regularization by virtue of the said CBA provisions.
121. Pacquing et. al. vs. Coca-Cola Phils.
[G.R. No. 157966. Jan. 31, 2008]
Facts:
Petitioners were sales route helpers or cargadores-pahinantes of Coca-Cola.
Petitioners sued for illegal dismissal and ULP. They claim that they are regular
employees. Coca-Cola claimed that petitioners were temporary workers who were
engaged for a five-month period to act as substitutes for an absent regular
employee.
Issue:
Whether petitioners are regular employees
Held:
Applying the principle stare decisis et non quieta movere (follow past precedents
and do not disturb what has been settled), SC applied the ruling in Magsalin v.
National Organization of Working Men to this case.
The basic law on the case is Article 280 (LCP). In determining whether an
employment should be considered regular or non-regular, the applicable test is the
reasonable connection between the particular activity performed by the employee
in relation to the usual business or trade of the employer. The standard, supplied
by the law itself, is whether the work undertaken is necessary or desirable in the
usual business or trade of the employer, a fact that can be assessed by looking into
the nature of the services rendered and its relation to the general scheme under
which the business or trade is pursued in the usual course. It is distinguished from a
specific undertaking that is divorced from the normal activities required in carrying
on the particular business or trade.
But, although the work to be performed is only for a specific project or
seasonal, where a person thus engaged has been performing the job for at least
28

one year, even if the performance is not continuous or is merely intermittent, the
law deems the repeated and continuing need for its performance as being
sufficient to indicate the necessity or desirability of that activity to the business or
trade of the employer. The employment of such person is also then deemed to be
regular with respect to such activity and while such activity exists.
The argument of petitioner that its usual business or trade is softdrink
manufacturing and that the work assigned to respondent workers as sales route
helpers so involves merely post production activities, one which is not
indispensable in the manufacture of its products, scarcely can be persuasive. If, as
so argued by petitioner company, only those whose work are directly involved in
the production of softdrinks may be held performing functions necessary and
desirable in its usual business or trade, there would have then been no need for it
to even maintain regular truck sales route helpers. The nature of the performed
must be viewed from a perspective of the business or trade in its entirety and not
on a confined scope.
The repeated rehiring of respondent workers and the continuing need for
their services clearly attest to the necessity or desirability of their services in the
regular conduct of the business or trade of petitioner company. Being regular
employees of respondent, petitioners are entitled to security of tenure, as provided
in Article 279 of the Labor Code, and may only be terminated from employment
due to just or authorized causes. Because respondent failed to show such cause,
the petitioners are deemed illegally dismissed and therefore entitled to back wages
and reinstatement without loss of se
122. COCOMANGAS HOTEL BEACH RESORT and/or MUNRO v. VISCA
Facts
Respondents alleged that they were regular employees of petitioners, who worked
as foreman, carpenters and masons tasked with the maintenance and repair of the
resort facilities. The hotels Front Desk Officer/Sales Manager informed them not to
report for work since the ongoing constructions and repairs would be temporarily
suspended as they cause irritation and annoyance to the resorts guests. As
instructed, they did not report for work the succeeding days. John, husband of
petitioner Munro, subsequently visited respondent foreman Visca and informed
him that the work suspension was due to budgetary constraints. However,
respondent Visca later discovered that 4 new workers were hired to do their tasks.
He confronted petitioner Munro who explained that respondents resumption of
work was not possible due to budgetary constraints. However, not less than 10
workers were subsequently hired by petitioners to do repairs in 2 cottages of the
resort. Two workers were also retained after completion.
Respondents thus filed their individual complaints for illegal dismissal, praying for
reinstatement with payment of full backwages, premium pay for rest day, service
incentive leave pay, 13
th
month pay, cost-of-living allowance, plus moral and
exemplary damages and attorneys fees.
Petitioners denied any employment relationship with respondents and countered
that respondent Visca was an independent contractor who was called upon from
time to time when some repairs in the resort facilities were needed and the other
respondents were selected and hired by him.
The LA dismissed the complaint, giving more credence to petitioners arguments.
Respondents filed a Memorandum of Appeal. The NLRC rendered a Decision setting
aside the LA decision, ordering the payment to respondents of backwages
computed from May 8, 1999 to July 31, 2002, 13
th
month pay and service incentive
leave pay for three years, in addition to 10% attorneys fees. Petitioners filed an
MR, from which the NLRC made a complete turnabout from its original decision and
issued a Resolution dismissing the complaint. Respondents then filed a Petition for
Certiorari with the CA. The CA rendered its Decision, reinstating the first decision of
the NLRC. Petitioners filed an MR, but it was denied by the CA.
Issues and Ruling
Are respondents regular or project employees? (It follows that if they are regular
employees, then they were illegally dismissed.)
REGULAR EMPLOYEES.
A project employee is one whose employment has been fixed for a specific project
or undertaking, the completion or termination of which has been determined at the
time of the engagement of the employee or where the work or service to be
performed is seasonal in nature and the employment is for the duration of the
season. Before an employee hired on a per-project basis can be dismissed, a report
must be made to the nearest employment office, of the termination of the services
of the workers every time completes a project, pursuant to Policy Instruction No.
20.
Respondents cannot be considered as project employees since they worked
continuously for petitioners from 3 to 12 years without any mention of a project
to which they were specifically assigned. While they had designations as foreman,
29

carpenter and mason, they performed work other than carpentry or masonry.
They were tasked with the maintenance and repair of the furniture, motor boats,
cottages, and windbreakers and other resort facilities. There is likewise no evidence
of the project employment contracts covering respondents alleged periods of
employment. More importantly, there is no evidence that petitioners reported the
termination of respondents supposed project employment to the DOLE as project
employees. Department Order No. 19, as well as the old Policy Instructions No. 20,
requires employers to submit a report of an employees termination to the nearest
public employment office every time his employment is terminated due to a
completion of a project.
In Maraguinot, Jr. v. National Labor Relations Commission, the Court ruled that
once a project or work pool employee has been: 1) continuously, as opposed to
intermittently, rehired by the same employer for the same tasks or nature of tasks;
and 2) these tasks are vital, necessary and indispensable to the usual business or
trade of the employer, then the employee must be deemed a regular employee,
pursuant to Article 280 of the Labor Code and jurisprudence.
What should be the reckoning period of the computation of the respondents
backwages?
Article 279 of the Labor Code, as amended, provides that an illegally dismissed
employee shall be entitled to reinstatement, full backwages, inclusive of
allowances, and to his other benefits or their monetary equivalent computed from
the time his compensation was withheld from him up to the time of his actual
reinstatement.
The NLRC, in its earlier Decision, computed the award for backwages from May 8,
1999 to July 31, 2002 only. It is evident that respondents backwages should not be
limited to said period. The backwages due respondents must be computed from the
time they were unjustly dismissed until actual reinstatement to their former
positions. Thus, until petitioners implement the reinstatement aspect, its obligation
to respondents, insofar as accrued backwages and other benefits are concerned,
continues to accumulate.
123. Price v. Innodata Philippines, Inc.
G.R. No. 178505, September 30, 2008
Facts:
Respondent Innodata was a corporation engaged in the data encoding and
conversion business. Petitioners-employees were encoders, indexers, formatters,
programmers, quality/quantity staff, etc.
The parties entered into a contract denominated as a Contract of
Employment for a Fixed Period stipulating that the contract shall be for a period of
one year effective February 16, 1999 to February 16, 2000. Furthermore, the
contract stipulates that should Innodata have no more need for the employees
services on account of completion of the project, lack of work due to business
losses, introduction of new production processes and techniques, which will negate
the need for personnel, and/or overstaffing, the contract maybe pre-terminated by
the Innodata upon giving of 3 days-notice to the employee. Moreover, it stipulates
that the employee or the Innodata may pre-terminate the contract, with or without
cause, by giving at least 15 days-notice to that effect.
On February 16, 2000, Innodata wrote petitioners informing them of the
cessation of their employment due to the expiration of the contract. As a result,
petitioner filed a complaint for illegal dismissal.
Petitioners contentions: 1) They should be considered regular employees
since their positions as formatters were necessary and desirable to the usual
business of Innodata as an encoding, conversion and data processing company; 2)
They could not be considered project employees considering that their employment
was not coterminous with any project or undertaking, the termination of which was
predetermined.
Innodatas contentions: 1) Petitioners were not illegally dismissed, for their
employment was terminated due to the expiration of their terms of employment.
Petitioners contracts of employment with Innodata were for a limited period only,
commencing on 6 September 1999 and ending on 16 February 2000; 2) Petitioners
were estopped from asserting a position contrary to the contracts which they had
knowingly, voluntarily, and willfully agreed to or entered into; 3) Petitioners were
project employees whose employment ceased at the end of a specific project or
undertaking.
Issues:
1. Were the petitioners regular employees of Innodata?
2. Were the petitioners hired under valid fixed-term employment contracts?
3. Were petitioners project employees?
Ruling:
1. YES. According to Article 280 of the Labor Code, the following employees
are accorded regular status: 1) those who are engaged to perform
30

activities which are necessary or desirable in the usual business or trade of
the employer, regardless of the length of their employment; and 2) those
who were initially hired as casual employees, but have rendered at least
one year of service, whether continuous or broken, with respect to the
activity in which they are employed.
Petitioners belong to the first type of employees. The applicable
test to determine whether an employment should be considered regular or
non-regular is the reasonable connection between the particular activity
performed by the employee in relation to the usual business or trade of
the employer. In the case at bar, petitioners were employed by Innodata as
formatters. The primary business of INNODATA is data encoding, and the
formatting of the data entered into the computers is an essential part of
the process of data encoding. Formatting organizes the data encoded,
making it easier to understand for the clients and/or the intended end
users thereof. Undeniably, the work performed by petitioners was
necessary or desirable in the business or trade of Innodata.
2. NO. While the Court has recognized the validity of fixed-term employment
contracts, it has consistently held that this is the exception rather than the
general rule.
Where, from the circumstances, it is apparent that the period was
imposed to preclude the acquisition of tenurial security by the employee,
then it should be struck down as being contrary to law, morals, good
customs, public order and public policy. After considering petitioners
contracts in their entirety, as well as the circumstances surrounding
petitioners employment, the Court is convinced that the terms fixed
therein were meant only to circumvent petitioners right to security of
tenure and are, therefore, invalid.
Petitioners alleged that their employment contracts became effective
February 16, 1999, and the first day they reported for work was on 17
February 1999. However, respondents asserted before the Labor Arbiter
that petitioners employment contracts were effective only on September
6, 1999. Innodata admitted that petitioners were originally hired on
February 16, 1999 but the project for which they were employed was
completed before the expiration of one year. Petitioners were merely
rehired on September 6, 1999 for a new project.
The Court notes that the attempt to change the beginning date of
effectivity of petitioners contracts was very crudely done. Such
modification and denial by respondents as to the real beginning date of
petitioners employment contracts render the said contracts ambiguous. If
the contracts took effect only on September 6, 1999, then its period of
effectivity would obviously be less than one year, or for a period of only
about five months. Obviously, respondents wanted to make it appear that
petitioners worked for Innodata for a period of less than one year.
3. NO. Project employees are those workers hired 1) for a specific project or
undertaking, and wherein 2) the completion or termination of such project
has been determined at the time of the engagement of the employee.
The employment contracts failed to reveal any mention therein of what
specific project or undertaking petitioners were hired for. Although the
contracts made general references to a "project," such project was neither
named nor described at all therein. The one-year period for which
petitioners were hired was simply fixed in the employment contracts
without reference or connection to the period required for the completion
of a project.
*As a final observation, petitioners have no right at all to expect security of
tenure, even for the supposedly one-year period of employment provided
in their contracts, because they can still be pre-terminated 1) upon the
completion of an unspecified project; or 2) with or without cause, for as
long as they are given a three-day notice. Such contract provisions are
repugnant to the basic tenet in labor law that no employee may be
terminated except for just or authorized cause.

124. Agusan Del Norte Electric Cooperative vs. Cagampang
G.R. No. 167627, October 10, 2008

Joel Cagampang and Glenn Garzon started working as linemen for Agusan del Norte
Electric Cooperative (ANECO) in 1990, under an employment contract for a period
not exceeding 3 months. They were required to work 8 hours a day and sometimes
on Sundays, with a daily salary of P122.00. When the contract expired, the two
were laid-off for one to five days and then ordered to report back to work but on
the basis of job orders. Their job orders, with employment periods of about 3
months each, were renewed several times, until 1998 & 1999 when such contracts
were no longer renewed, resulting in their loss of employment. In 2001, they filed a
case for illegal dismissal against ANECO, and prayed for payment of backwages,
salary differential, allowances, premium for alleged work during holidays and rest
days, service incentive leave, and separation pay.

The LA declared the dismissal illegal and ordered ANECO to pay their money claims.
The NLRC set aside the LA decision except the portions granting service incentive
31

leave pay; attorney's fees, fixed at ten percent (10%) of the total money award to
both respondents; and salary differential to Garzon. The CA reinstated the LA
decision.

ISSUE: WON the respondents were illegally dismissed.

RULING: Yes. The test to determine whether employment is regular or not is the
reasonable connection between the particular activity performed by the employee
in relation to the usual business or trade of the employer. Also, if the employee has
been performing the job for at least one year, even if the performance is not
continuous or merely intermittent, the law deems the repeated and continuing
need for its performance as sufficient evidence of the necessity, if not
indispensability of that activity to the business. Thus, we held that where the
employment of project employees is extended long after the supposed project has
been finished, the employees are removed from the scope of project employees
and are considered regular employees.

Respondents in the present case being regular employees, ANECO as the employer
had the burden of proof to show that the respondents' termination was for a just
cause. Unfortunately, however, what petitioners did was merely to refuse, without
justifiable reason, to renew respondents' work contracts for the performance of
what would otherwise be regular jobs in relation to the trade or business of the
former. Such conduct dismally falls short of the requirements of our labor laws
regarding dismissals. No twin notices of termination were issued to the employees,
hence the employer did not observe due process in dismissing them from their
employment. Their dismissals were patently illegal.
125. WILLIAM UY CONSTRUCTION CORP. and/or TERESITA UY and WILLIAM UY vs.
JORGE R. TRINIDAD
G.R. No. 183250. March 12, 2010.*
Facts:
Respondent Trinidad filed a complaint for illegal dismissal and unpaid
benefits against petitioner William Uy Construction Corporation. Respondent
claimed that he had been working with petitioner company for 16 years since 1988
as driver of its service vehicle, dump truck, and transit mixer. He had signed several
employment contracts with the company that identified him as a project employee
although he had always been assigned to work on one project after another with
some intervals. Although the company had shut down operation for a while, but
when it opened its project in batangas, it did not hire back respondent.
On the other hand, Petitioner company countered that it was in the
construction business. By the nature of such business, it had to hire and engage the
services of project construction workers, including respondent Trinidad, whose
employments had to be coterminous with the completion of specific company
projects. For this reason, every time the company employed Trinidad, he had to
execute an employment contract, Appointment as Project Worker.
The LA dismissed respondents complaint and held that the latter was a
project employee and since his company submitted the appropriate establishment
termination report to DOLE, his loss of work cannot be regarded as unjust dismissal.
This decision was affirmed by the NLRC. However, it was reversed by the CA.
Issue:
Whether or not petitioner companys repeated rehiring of respondent
Trinidad over several years as project employee for its various projects
automatically entitled him to the status of a regular employee.
Ruling:
NO. The test for distinguishing a project employee from a regular
employee is whether or not he has been assigned to carry out a specific project or
undertaking, with the duration and scope of his engagement specified at the time
his service is contracted. Here, it is not disputed that petitioner company
contracted respondent Trinidads service by specific projects with the duration of
his work clearly set out in his employment contracts. He remained a project
employee regardless of the number of years and the various projects he worked for
the company.
Generally, length of service provides a fair yardstick for determining when
an employee initially hired on a temporary basis becomes a permanent one,
entitled to the security and benefits of regularization. But this standard will not be
fair, if applied to the construction industry, simply because construction firms
cannot guarantee work and funding for its payrolls beyond the life of each project.
And getting projects is not a matter of course. Construction companies have no
control over the decisions and resources of project proponents or owners. There is
no construction company that does not wish it has such control but the reality,
understood by construction workers, is that work depended on decisions and
developments over which construction companies have no say.
In Caseres v. Universal Robina Sugar Milling Corporation, the repeated and
successive rehiring of project employees do not qualify them as regular
32

employees, as length of service is not the controlling determinant of the
employment tenure of a project employee, but whether the employment
has been fixed for a specific project or undertaking, its completion has
been determined at the time of the engagement of the employee.
In this case, respondent Trinidads series of employments with petitioner
company were co-terminous with its projects. He was not dismissed. His
employment contract simply ended with the project for which he had signed up.
His employment history belies the claim that he continuously worked for the
company. Intervals or gaps separated one contract from another.
126. Dacuital vs. L.M. Camus Engineering Corp.,
G.R. No. 176748, September 1, 2010
Facts:
Respondent L.M. Camus Engineering Corporation (LMCEC) is a domestic
corporation engaged in construction, engineering, and air-conditioning business;
while respondent Luis M. Camus (Camus) is the company president. Petitioners
were hired by LMCEC as welder, tinsmith, pipefitter, and mechanical employees.
Petitioners filed a complaint for illegal dismissal and non-payment of monetary
benefits against respondent LM Camus Engineering Corp. before the National Labor
Relations Commission (NLRC). The employees alleged that they were illegally
dismissed from employment and that their employer failed to pay them their
holiday pay, premium pay for holiday, rest day, service incentive leave pay, and
13th month pay during the existence and duration of their employment. They also
averred that they were not provided with sick and vacation leaves.
Respondents denied that petitioners were illegally dismissed from employment.
They claimed that petitioners were project employees and, upon the completion of
each project, they were served notices of project completion. They clarified that the
termination of petitioners employment was due to the completion of the projects
for which they were hired.
Petitioners, however, countered that they were regular employees as they
had been engaged to perform activities which are usually necessary or desirable in
the usual business or trade of LMCEC. They denied that they were project or
contractual employees because their employment was continuous and
uninterrupted for more than one (1) year. Finally, they maintained that they were
part of a work pool from which LMCEC drew its workers for its various projects.
The Labor Arbiter rendered a decision declaring the dismissal of the
complainant-employees as ILLEGAL and the complainants are entitled to
reinstatement without back wages. The NLRC modified the decision of the Labor
Arbiter and ordered the reinstatement of the complainants with limited backwages.
The respondents appealed the decision to the Court of Appeals and the appellate
court held that the complainants are PROJECT EMPLOYEES and hence, there was no
illegal dismissal.

ISSUE:

1. Whether or not the petitioners were project employees.
2. Whether or not their dismissal from employment was legal.

RULING:
1. No, the petitioners were not project employees but regular employees.

Article 280 of the Labor Code distinguishes a "project employee" from a
"regular employee" in this wise:

Article 280. Regular and casual employment.The provisions of
written agreement to the contrary notwithstanding and regardless of the
oral agreement of the parties, an employment shall be deemed to be
regular where the employee has been engaged to perform activities which
are usually necessary or desirable in the usual business or trade of the
employer, except where the employment has been fixed for a specific
project or undertaking the completion or termination of which has been
determined at the time of the engagement of the employee or where the
work or services to be performed is seasonal in nature and the
employment is for the duration of the season...xxx

The principal test used to determine whether employees are project
employees is whether or not the employees were assigned to carry out a specific
project or undertaking, the duration or scope of which was specified at the time the
employees were engaged for that project.
Even though the absence of a written contract does not by itself grant
regular status to petitioners, such a contract is evidence that petitioners were
informed of the duration and scope of their work and their status as project
employees. In this case, where no other evidence was offered, the absence of the
employment contracts raises a serious question of whether the employees were
properly informed at the onset of their employment of their status as project
employees.
33

While it is true that respondents presented the employment contract of
Dacuital, the contract does not show that he was informed of the nature, as well as
the duration of his employment. In fact, the duration of the project for which he
was allegedly hired was not specified in the contract. Furthermore, even if we
assume that under the above provision of the contract, Dacuital was informed of
the nature of his employment and the duration of the project, that same contract is
not sufficient evidence to show that the other employees were so informed. It is
undisputed that petitioners had individual employment contracts, yet respondents
opted not to present them on the lame excuse that they were similarly situated as
Dacuital. The non-presentation of these contracts gives rise to the presumption that
the employees were not informed of the nature and duration of their employment.
It is doctrinally entrenched that in illegal dismissal cases, the employer has the
burden of proving with clear, accurate, consistent, and convincing evidence that the
dismissal was valid. Absent any other proof that the project employees were
informed of their status as such, it will be presumed that they are regular
employees.
Moreover, Department Order No. 19 (as well as the old Policy Instructions
No. 20) requires employers to submit a report of an employees termination to the
nearest public employment office everytime the employment is terminated due to
the completion of a project. In this case, there was no evidence that there was
indeed such a report. LMCECs failure to file termination reports upon the cessation
of petitioners employment was an indication that petitioners were not project but
regular employees.
2. No, the dismissal was illegal. Because they were regular employees, they
enjoy security of tenure and they can only be dismissed for just or valid
cause and upon compliance with due process, i.e., after notice and hearing.
In cases involving an employees dismissal, the burden is on the employer
to prove that the dismissal was legal. This burden was not amply
discharged by LMCEC in this case. Being regular employees, petitioners
were entitled to security of tenure, and their services may not be
terminated except for causes provided by law.
Furthermore, records failed to show that LMCEC afforded petitioners, as
regular employees, due process prior to their dismissal, through the twin
requirements of notice and hearing. Petitioners were not served notices informing
them of the particular acts for which their dismissal was sought. Nor were they
required to give their side regarding the charges made against them, if any.
Certainly, petitioners dismissal was not carried out in accordance with law and was,
therefore, illegal.
127. MILLENNIUM ERECTORS CORPORATION , vs. MAGALLANES
G.R. No. 184362. November 15, 2010.*
FACTS: Respondent VirgilioMagallanes started working in 1988 as a utility
man for Laurencito Tiu (Tiu), Chief Executive Officer of Millennium Erectors
Corporation (petitioner), Tius family, and Kenneth Construction Corporation. He
was assigned to different construction projects undertaken by petitioner in Metro
Manila, the last of which was for a building in Libis, Quezon City. In July of 2004 he
was told not to report for work anymore allegedly due to old age, prompting him to
file an illegal dismissal complaint before the Labor Arbiter.
Petitioner claimed that respondent was a project employee whom it hired
for a building project in Libis on January 30, 2003, that on August 3, 2004,
respondents services were terminated as the project was nearing completion; and
he was given financial assistance in the amount of P2,000, for which he signed a
quitclaim and waiver. Petitioner likewise submitted a termination report to the
Department of Labor and Employment (DOLE).
LAS RULING: Labor Arbiter ruled in favor of petitioner and dismissed the complaint,
holding that respondent knew of the nature of his employment as a project
employee, he having executed an employment contract specifying therein the
name of and duration of the project from January 2003 until its completion.
NLRCS RULING: On appeal, the National Labor Relations Commission (NLRC) set
aside the Labor Arbiters Decision. It held that respondent was a regular, not a
project employee, as the employment contract he supposedly signed contained the
date of commencement but not a specific date when it would end and that based
on the payrolls petitioner submitted and contrary to its claim that respondent was
hired 2003, he had been employed in 2001, not 2003, lending weight to his claim
that he had worked for petitioner for 16 years prior to the filing of his complaint.
CAs RULING: It affirmed the nlrcs decision.
ISSUE: what is the status of employment of the respondent?
RULING: Respondent was a regular, not a project employee.
As defined in the case of Saberola v. Suarez: A project employee is one whose
"employment has been fixed for a specific project or undertaking, the completion
or termination of which has been determined at the time of the engagement of
34

the employee or where the work or service to be performed is seasonal in nature
and the employment is for the duration of the season." And as held in the case of
Equipment Technical Services v. Court of Appeals, the difference between a regular
employee and a project employee is that the service of project employees are
coterminus with the project and may be terminated upon the end or completion of
that project or project phase for which they were hired. Regular employees, in
contrast, enjoy security of tenure and are entitled to hold on to their work or
position until their services are terminated by any of the modes recognized under
the Labor Code.
Petitioners various payrolls dating as early as 2001 show that respondent
had been employed by it. As aptly observed by the appellate court, these
documents, rather than sustaining petitioners argument, only serve to support
respondents contention that he had been employed in various projects, if not for
16 years, at the very least two years prior to his dismissal. Assuming arguendo that
petitioner hired respondent initially on a per project basis, his continued rehiring, as
shown by the sample payrolls converted his status to that of a regular employee.
Following Cocomangas Beach Hotel Resort v. Visca, the repeated and continuing
need for respondents services is sufficient evidence of the necessity, if not
indispensability, of his services to petitioner's business and, as a regular employee,
he could only be dismissed from employment for a just or authorized cause.
Petitioner having failed to discharge its burden of proving that it
terminated the services of respondent for cause and with due process, the
challenged decision must remain.
128. EXODUS vs BISCOCHO
FACTS: Petitioner Exodus International Construction Corporation (Exodus) is a duly
licensed labor contractor for the painting of residential houses, condominium units
and commercial buildings. Petitioner Antonio P. Javalera is the President and
General Manager of Exodus. On February 1, 1999, Exodus obtained from Dutch Boy
Philippines, Inc. (Dutch Boy) a contract
6
for the painting of the Imperial Sky Garden
located at Ongpin Street, Binondo, Manila. On July 28, 1999, Dutch Boy awarded
another contract
7
to Exodus for the painting of Pacific Plaza Towers in Fort
Bonifacio, Taguig City.
In the furtherance of its business, Exodus hired respondents as painters on different
dates with the corresponding wages. On November 27, 2000, Guillermo, Fernando,
Ferdinand, and Miguel filed a complaint
8
for illegal dismissal and non-payment of
holiday pay, service incentive leave pay, 13th month pay and night-shift differential
pay. On December 1, 2000, Gregorio also filed a complaint
9
. He claimed that he was
dismissed from the service on September 12, 2000 while Guillermo, Fernando,
Ferdinand, and Miguel were orally notified of their dismissal from the service on
November 25, 2000.
Petitioners denied respondents allegations. As regards Gregorio, petitioners
averred that on September 15, 2000, he absented himself from work and applied as
a painter with SAEI-EEI which is the general building contractor of Pacific Plaza
Towers. Since then, he never reported back to work. Guillermo absented himself
from work without leave on November 27, 2000. When he reported for work the
following day, he was reprimanded for being Absent Without Official Leave (AWOL).
Because of the reprimand, he worked only half-day and thereafter was unheard of
until the filing of the instant complaint. Fernando, Ferdinand, and Miguel were
caught eating during working hours on November 25, 2000 for which they were
reprimanded by their foreman. Since then they no longer reported for work.
1. Whether there was dismissal in this case.
NO. As found by the Labor Arbiter, there was no evidence that respondents were
dismissed nor were they prevented from returning to their work. It was only
respondents unsubstantiated conclusion that they were dismissed. As a matter of
fact, respondents could not name the particular person who effected their dismissal
and under what particular circumstances.
In Machica v. Roosevelt Services Center, Inc.,
23
this Court sustained the employer's
denial as against the employees' categorical assertion of illegal dismissal. In so
ruling, this Court held that:
The rule is that one who alleges a fact has the burden of proving it; thus, petitioners
were burdened to prove their allegation that respondents dismissed them from
their employment. It must be stressed that the evidence to prove this fact must be
clear, positive and convincing. The rule that the employer bears the burden of proof
in illegal dismissal cases finds no application here because the respondents deny
having dismissed the petitioners.
In this case, petitioners were able to show that they never dismissed respondents.
As to the case of Fernando, Miguel and Ferdinand, it was shown that on November
25, 2000, at around 7:30 a.m., the petitioners foreman, Wenifredo Lalap
(Wenifredo) caught the three still eating when they were supposed to be working
already. Wenifredo reprimanded them and, apparently, they resented it so they no
35

longer reported for work. In the case of Gregorio, he absented himself from work
on September 15, 2000 to apply as a painter with SAEI-EEI, the general contractor
of Pacific Plaza Towers. Since then he never reported back to work. Lastly, in the
case of Guillermo, he absented himself without leave on November 27, 2000, and
so he was reprimanded when he reported for work the following day. Because of
the reprimand, he did not report for work anymore.
Hence, as between respondents general allegation of having been orally dismissed
from the service vis-a-vis those of petitioners which were found to be substantiated
by the sworn statement of foreman Wenifredo, we are persuaded by the latter.
Absent any showing of an overt or positive act proving that petitioners had
dismissed respondents, the latters claim of illegal dismissal cannot be sustained.
Indeed, a cursory examination of the records reveal no illegal dismissal to speak of.
2. Was there abandonment of work on the part of the respondents?
NO. The Labor Arbiter is also correct in ruling that there was no abandonment on
the part of respondents that would justify their dismissal from their employment.
It is a settled rule that "[m]ere absence or failure to report for work x x x is not
enough to amount to abandonment of work."
24
"Abandonment is the deliberate
and unjustified refusal of an employee to resume his employment."
25

In Northwest Tourism Corporation v. Former Special 3rd Division of the Court of
Appeals
26
this Court held that "[t]o constitute abandonment of work, two elements
must concur, [namely]:
(1) the employee must have failed to report for work or must have been
absent without valid or justifiable reason; and
(2) there must have been a clear intention on the part of the employee to
sever the employer-employee relationship manifested by some overt act."
"It is the employer who has the burden of proof to show a deliberate and
unjustified refusal of the employee to resume his employment without any
intention of returning."
27
It is therefore incumbent upon petitioners to ascertain the
respondents interest or non-interest in the continuance of their employment.
However, petitioners failed to do so.
3. Petitioners are of the position that the reinstatement of respondents to
their former positions, which were no longer existing, is impossible,
highly unfair and unjust. The project was already completed by
petitioners on September 28, 2001. Thus the completion of the project left
them with no more work to do. Having completed their tasks, their
positions automatically ceased to exist. Consequently, there were no
more positions where they can be reinstated as painters. Is this tenable?
NO. Petitioners are misguided. They forgot that there are two types of employees in
the construction industry. The first is referred to as project employees or those
employed in connection with a particular construction project or phase thereof and
such employment is coterminous with each project or phase of the project to which
they are assigned. The second is known as non-project employees or those
employed without reference to any particular construction project or phase of a
project. The second category is where respondents are classified. As such they are
regular employees of petitioners. It is clear from the records of the case that when
one project is completed, respondents were automatically transferred to the next
project awarded to petitioners. There was no employment agreement given to
respondents which clearly spelled out the duration of their employment, the
specific work to be performed and that such is made clear to them at the time of
hiring. It is now too late for petitioners to claim that respondents are project
employees whose employment is coterminous with each project or phase of the
project to which they are assigned.
Nonetheless, assuming that respondents were initially hired as project employees,
petitioners must be reminded of our ruling in Maraguinot, Jr. v. National Labor
Relations Commission
28
that "[a] project employee x x x may acquire the status of a
regular employee when the following [factors] concur:
1. There is a continuous rehiring of project employees even after cessation
of a project; and
2. The tasks performed by the alleged "project employee" are vital,
necessary and indespensable to the usual business or trade of the
employer."
In this case, the evidence on record shows that respondents were employed and
assigned continuously to the various projects of petitioners. As painters, they
performed activities which were necessary and desirable in the usual business of
petitioners, who are engaged in subcontracting jobs for painting of residential units,
condominium and commercial buildings.
36

129. LEYTE GEOTHERMAL vs PNOC EDC
FACTS: [PNOC-EDC] is a government-owned and controlled corporation engaged in
exploration, development, utilization, generation and distribution of energy
resources like geothermal energy. Petitioner is a legitimate labor organization. The
[respondent] hired and employed hundreds of employees on a contractual basis,
whereby, their employment was only good up to the completion or termination of
the project and would automatically expire upon the completion of such project.
Majority of the employees hired by [respondent] in its Leyte Geothermal Power
Projects had become members of petitioner. In view of that circumstance, the
petitioner demands from the [respondent] for recognition of it as the collective
bargaining agent of said employees and for a CBA negotiation with it. However, the
[respondent] did not heed such demands of the petitioner. Sometime in 1998 when
the project was about to be completed, the [respondent] proceeded to serve
Notices of Termination of Employment upon the employees who are members of
the petitioner. On December 28, 1998, the petitioner filed a Notice of Strike with
DOLE against the [respondent] on the ground of purported commission by the
latter of unfair labor practice for "refusal to bargain collectively, union busting and
mass termination." On the same day, the petitioner declared a strike and staged
such strike.
To avert any work stoppage, then Secretary of Labor Bienvenido E. Laguesma
intervened and issued the Order, dated January 4, 1999, certifying the labor dispute
to the NLRC for compulsory arbitration. Accordingly, all the striking workers were
directed to return to work within twelve (12) hours from receipt of the Order and
for the [respondent] to accept them back under the same terms and conditions of
employment prior to the strike. Further, the parties were directed to cease and
desist from committing any act that would exacerbate the situation. However,
despite earnest efforts on the part of the Secretary of Labor and Employment to
settle the dispute amicably, the petitioner remained adamant and unreasonable in
its position, causing the failure of the negotiation towards a peaceful compromise.
In effect, the petitioner did not abide by [the] assumption order issued by the
Secretary of Labor. Consequently, on January 15, 1999, the [respondent] filed a
Complaint for Strike Illegality, Declaration of Loss of Employment and Damages at
the NLRC-RAB VIII in Tacloban City and at the same time, filed a Petition for
Cancellation of Petitioners Certificate of Registration with DOLE.
1. Whether the officers and members of petitioner Union are project
employees of respondent.
YES. In the case at bar, the records reveal that the officers and the members of
petitioner Union signed employment contracts indicating the specific project or
phase of work for which they were hired, with a fixed period of employment. The
NLRC correctly disposed of this issue:
A deeper examination also shows that [the individual members of petitioner Union]
indeed signed and accepted the [employment contracts] freely and voluntarily. No
evidence was presented by [petitioner] Union to prove improper pressure or undue
influence when they entered, perfected and consummated [the employment]
contracts. In fact, it was clearly established in the course of the trial of this case, as
explained by no less than the President of [petitioner] Union, that the contracts of
employment were read, comprehended, and voluntarily accepted by them. x x x.
x x x x
As clearly shown by *petitioner+ Unions own admission, both parties had executed
the contracts freely and voluntarily without force, duress or acts tending to vitiate
the worker*s+ consent. Thus, we see no reason not to honor and give effect to the
terms and conditions stipulated therein. x x x.
13

Thus, we are hard pressed to find cause to disturb the findings of the NLRC which
are supported by substantial evidence.
In the case at bar, both the NLRC and the CA were one in the conclusion that the
officers and the members of petitioner Union were project employees.
Nonetheless, petitioner Union insists that they were regular employees since they
performed work which was usually necessary or desirable to the usual business or
trade of the Construction Department of respondent.
The landmark case of ALU-TUCP v. NLRC
17
instructs on the two (2) categories of
project employees:
It is evidently important to become clear about the meaning and scope of the term
"project" in the present context. The "project" for the carrying out of which "project
employees" are hired would ordinarily have some relationship to the usual business
of the employer. Exceptionally, the "project" undertaking might not have an
ordinary or normal relationship to the usual business of the employer. In this latter
case, the determination of the scope and parameters of the "project" becomes
fairly easy. x x x. From the viewpoint, however, of the legal characterization
problem here presented to the Court, there should be no difficulty in designating
the employees who are retained or hired for the purpose of undertaking fish
culture or the production of vegetables as "project employees," as distinguished
from ordinary or "regular employees," so long as the duration and scope of the
project were determined or specified at the time of engagement of the "project
37

employees." For, as is evident from the provisions of Article 280 of the Labor Code,
quoted earlier, the principal test for determining whether particular employees are
properly characterized as "project employees" as distinguished from "regular
employees," is whether or not the "project employees" were assigned to carry out a
"specific project or undertaking," the duration (and scope) of which were specified
at the time the employees were engaged for that project.
Plainly, the litmus test to determine whether an individual is a project employee lies
in setting a fixed period of employment involving a specific undertaking which
completion or termination has been determined at the time of the particular
employees engagement. In this case, as previously adverted to, the officers and the
members of petitioner Union were specifically hired as project employees for
respondents Leyte Geothermal Power Project located at the Greater Tongonan
Geothermal Reservation in Leyte. Consequently, upon the completion of the project
or substantial phase thereof, the officers and the members of petitioner Union
could be validly terminated.
2. Petitioner Union is adamant, however, that the lack of interval in the
employment contracts of its officer and members negates the latters
status as mere project employees. For petitioner Union, the lack of
interval further drives home its point that its officers and members are
regular employees who performed work which was usually necessary or
desirable to the usual business or trade of respondent.
We are not persuaded. Petitioner Unions members employment for more than a
year does equate to their regular employment with respondent. In this regard,
Mercado, Sr. v. NLRC
19
illuminates:
The first paragraph [of Article 280 of the Labor Code] answers the question of who
are regular employees. It states that, regardless of any written or oral agreement to
the contrary, an employee is deemed regular where he is engaged in necessary or
desirable activities in the usual business or trade of the employer, except for project
employees.
A project employee has been defined to be one whose employment has been fixed
for a specific project or undertaking, the completion or termination of which has
been determined at the time of the engagement of the employee, or where the
work or service to be performed is seasonal in nature and the employment is for
the duration of the season, as in the present case.
The second paragraph of Art. 280 demarcates as "casual" employees, all other
employees who do not fall under the definition of the preceding paragraph. The
proviso, in said second paragraph, deems as regular employees those "casual"
employees who have rendered at least one year of service regardless of the fact
that such service may be continuous or broken.
Petitioners, in effect, contend that the proviso in the second paragraph of Art. 280
is applicable to their case and that the Labor Arbiter should have considered them
regular by virtue of said proviso. The contention is without merit.
The general rule is that the office of a proviso is to qualify or modify only the phrase
immediately preceding it or restrain or limit the generality of the clause that it
immediately follows. Thus, it has been held that a proviso is to be construed with
reference to the immediately preceding part of the provision to which it is attached,
and not to the statute itself or to other sections thereof. The only exception to this
rule is where the clear legislative intent is to restrain or qualify not only the phrase
immediately preceding it (the proviso) but also earlier provisions of the statute or
even the statute itself as a whole.
Policy Instruction No. 12 of the Department of Labor and Employment discloses
that the concept of regular and casual employees was designed to put an end to
casual employment in regular jobs, which has been abused by many employers to
prevent so called casuals from enjoying the benefits of regular employees or to
prevent casuals from joining unions. The same instructions show that the proviso in
the second paragraph of Art. 280 was not designed to stifle small-scale businesses
nor to oppress agricultural land owners to further the interests of laborers, whether
agricultural or industrial. What it seeks to eliminate are abuses of employers against
their employees and not, as petitioners would have us believe, to prevent small-
scale businesses from engaging in legitimate methods to realize profit. Hence, the
proviso is applicable only to the employees who are deemed "casuals" but not to
the "project" employees nor the regular employees treated in paragraph one of Art.
280.
Clearly, therefore, petitioners being project employees, or, to use the correct term,
seasonal employees, their employment legally ends upon completion of the project
or the [end of the] season. The termination of their employment cannot and should
not constitute an illegal dismissal.
Considering our holding that the officers and the members of petitioner Union were
project employees, its claim of union busting is likewise dismissed.
130. ST. PAUL COLLEGE QUEZON CITY vs.. ANCHETA II
G.R. No. 169905, Sep. 07, 2011
38

Facts:
Petitioner St. Paul College, Quezon City (SPCQC) is a private Catholic
educational institution. Respondent Remigio Michael was hired by the SPCQC as a
teacher in the General Education Department with a probationary rank in the
School Year (SY) 1996-1997 which was renewed in the following SY 1997-1998. His
wife, respondent Cynthia was hired by the same school as a part time teacher of the
Mass Communication Department in the second semester of SY 1996-1997 and her
appointment was renewed for SY 1997-1998. Petitioner Sr. Bernadette, on March 9,
1998, sent two letter with the same contents to the respondent spouses informing
them that upon the recommendation of the College Council, the school is extending
to them new contracts for SY 1998-1999. On April 30, 1998, petitioner Sr.
Bernadette wrote a letter endorsement for the immediate termination of the
teaching services of the respondent spouses on the following grounds:
1. Non-compliance with the departmental policy to submit their
final test questions to their respective program coordinators for
checking/comments (violating par. 7.1, p. 65 of the Faculty
Manual).
This policy was formulated to ensure the validity and
reliability of test questions of teachers for the good of the
students. This in effect can minimize if not prevent unnecessary
failure of students.

2. Non-compliance with the standard format (multiple choice) of
final test questions as agreed upon in the department. Mr.
Ancheta prepared purely essay questions for the students.
Well-prepared multiple choice questions are more objective, and
develop critical thinking among students.

3. Failure to encode their modular grade reports as required
(violating par. H. 8, p. 66 of our Faculty manual).

4. Failure to submit and update required modules (syllabi) of
their subject despite reminders (violating D, 1.5, p. 40 of our
Faculty Manual).

5. Both spouses have a gross number of failure in their class.

Thus, respondent spouses filed a Complaint for illegal dismissal with the NLRC. On
November 20, 2000, the Labor Arbiter dismissed the complaint, the dispositive
portion of the decision reads:
The decision of the Labor Arbiter was appealed to the NLRC, but was affirmed
by the latter on February 28, 2003, disposing the case.
After the denial of their motion for reconsideration with the NLRC, the
respondent spouses filed a petition for certiorari with the CA. In its Decisiondated
July 8, 2005, the CA granted the petition and reversed the decisions of the Labor
Arbiter and the NLRC.

Issues:
(1) May a teacher on probationary employment automatically claim security
of tenure and compel the employer to renew his employment contract
after contract expiration?
(2) Were the Ancheta Spouses illegally dismissed?

Ruling:
(1) NO
A reality we have to face in the consideration of employment on
probationary status of teaching personnel is that they are not governed purely
by the Labor Code. The Labor Code is supplemented with respect to the
period of probation by special rules found in the Manual of Regulations for
Private Schools. On the matter of probationary period, Section 92 of these
regulations provides:

Section 92. Probationary Period
- Subject in all instances to compliance with the Department and
school requirements, the probationary period for academic personnel shall not
be more than three (3) consecutive years of satisfactory service for those in the
elementary and secondary levels, six (6) consecutive regular semesters of
satisfactory service for those in the tertiary level, and nine (9) consecutive
trimesters of satisfactory service for those in the tertiary level where collegiate
courses are offered on a trimester basis.
A probationary employee or probationer is one who is on trial for an
employer, during which the latter determines whether or not he is qualified
for permanent employment. The probationary employment is intended to
afford the employer an opportunity to observe the fitness of a probationary
employee while at work, and to ascertain whether he will become an efficient
and productive employee. While the employer observes the fitness, propriety
and efficiency of a probationer to ascertain whether he is qualified for
permanent employment, the probationer, on the other hand, seeks to prove
to the employer that he has the qualifications to meet the reasonable
standards for permanent employment. Thus, the word probationary, as used
to describe the period of employment, implies the purpose of the term or
period, not its length.
The common practice is for the employer and the teacher to enter
into a contract, effective for one school year. At the end of the school year,
39

the employer has the option not to renew the contract, particularly
considering the teacher's performance. If the contract is not renewed, the
employment relationship terminates. If the contract is renewed, usually for
another school year, the probationary employment continues. Again, at the
end of that period, the parties may opt to renew or not to renew the contract.
If renewed, this second renewal of the contract for another school year would
then be the last year since it would be the third school year of
probationary employment. At the end of this third year, the employer may
now decide whether to extend a permanent appointment to the employee,
primarily on the basis of the employee having met the reasonable standards
of competence and efficiency set by the employer. For the entire duration of
this three-year period, the teacher remains under probation. Upon the
expiration of his contract of employment, being simply on probation, he
cannot automatically claim security of tenure and compel the employer to
renew his employment contract.

(2) NO
The Labor Code commands that before an employer may legally
dismiss an employee from the service, the requirement of substantial and
procedural due process must be complied with. Under the requirement of
substantial due process, the grounds for termination of employment must
be based on just or authorized causes.
Petitioner school charged respondent Remigio Michael of non-
compliance with a school policy regarding the submission of final test
questions to his program coordinator for checking or comment. Following
due process, the same respondent admitted the charge in his letter. The
plain admissions of the charges against them were the considerations taken
into account by the petitioner school in their decision not to renew the
respondent spouses' employment contracts. This is a right of the school that
is mandated by law and jurisprudence. It is the prerogative of the school to
set high standards of efficiency for its teachers since quality education is a
mandate of the Constitution. As long as the standards fixed are reasonable
and not arbitrary, courts are not at liberty to set them aside. Schools cannot
be required to adopt standards which barely satisfy criteria set for
government recognition. The same academic freedom grants the school the
autonomy to decide for itself the terms and conditions for hiring its teacher,
subject of course to the overarching limitations under the Labor Code. The
authority to hire is likewise covered and protected by its management
prerogative the right of an employer to regulate all aspects of
employment, such as hiring, the freedom to prescribe work assignments,
working methods, process to be followed, regulation regarding transfer of
employees, supervision of their work, lay-off and discipline, and dismissal
and recall of workers.
131. Lynvil Fishing Enterprises vs. Ariola,
G.R. No. 181974, February 1, 2012
Facts:
Petitioner Lynvil Fishing Enterprises, Inc. (Lynvil) is engaged in deep-sea
fishing. Respondents services were engaged in various capacities: Andres G. Ariola,
captain; Jessie D. Alcovendas, chief mate; Jimmy B. Calinao, chief engineer; Ismael
G. Nubla, cook; Elorde Baez, oiler; and Leopoldo G. Sebullen, bodegero.
On Aug. 1, 1998, Lynvil received a report from Ramonito Clarido, one of its
employees, that on July 31, 1998, he witnessed that while on board the company
vessel Analyn VIII, respondents conspired with one another and stole eight tubs of
pampano and tangigue fish and delivered them to another vessel.
Petitioner filed a criminal complaint against respondents before the office
of the City Prosecutor of Malabon City which found probable cause for indictment
of respondents for the crime of qualified theft. Relying on the finding and Nasipit
Lumber Company v. NLRC, 257 Phil. 937 (1989), Lynvil asserted there was sufficient
basis for valid termination of employment of respondents based on serious
misconduct and/or loss of trust and confidence.

Issues:
Whether a finding of the city prosecutor of probable cause to indict
employees of qualified theft is sufficient basis for valid termination for serious
misconduct and/or loss of trust or confidence?
Whether the employees were validly terminated?
Whether De Borja is jointly and severally liable with Lynvil?

Ruling:
On the first issue, the Supreme Court ruled in the negative. We ruled that
proof beyond reasonable doubt of an employees misconduct is not required when
loss of confidence is the ground for dismissal. It is sufficient if the employer has
some basis to lose confidence or that the employer has reasonable ground to
believe or to entertain the moral conviction that the employee concerned is
responsible for the misconduct and that the nature of his participation therein
rendered him absolutely unworthy of the trust and confidence demanded by his
position.
Lynvil cannot argue that since the Office of the Prosecutor found probable
cause for theft the Labor Arbiter must follow the finding as a valid reason for the
termination of respondents employment. The proof required for purposes that
differ from one and the other are likewise different.
On the second question, the Court stated that nonetheless, even without
reliance on the prosecutors finding, we find that there was valid cause for
respondents dismissal.
40

Just cause is required for a valid dismissal. The Labor Code provides that
an employer may terminate an employment based on fraud or willful breach of the
trust reposed on the employee. Such breach is considered willful if it is done
intentionally, knowingly, and purposely, without justifiable excuse, as distinguished
from an act done carelessly, thoughtlessly, heedlessly or inadvertently. It must also
be based on substantial evidence and not on the employers whims or caprices or
suspicions otherwise, the employee would eternally remain at the mercy of the
employer. Loss of confidence must not be indiscriminately used as a shield by the
employer against a claim that the dismissal of an employee was arbitrary. And, in
order to constitute a just cause for dismissal, the act complained of must be work-
related and shows that the employee concerned is unfit to continue working for the
employer. In addition, loss of confidence as a just cause for termination of
employment is premised on the fact that the employee concerned holds a position
of responsibility, trust and confidence or that the employee concerned is entrusted
with confidence with respect to delicate matters, such as the handling or care and
protection of the property and assets of the employer. The betrayal of this trust is
the essence of the offense for which an employee is penalized. Breach of trust is
present in this case.
However, Lynvil contends that it cannot be guilty of illegal dismissal
because the private respondents were employed under a fixed-term contract which
expired at the end of the voyage. Contrarily, the private respondents (employees)
contend that they became regular employees by reason of their continuous hiring
and performance of tasks necessary and desirable in the usual trade and business of
Lynvil.
Jurisprudence, laid two conditions for the validity of a fixed-contract
agreement between the employer and employee: first, the fixed period of
employment was knowingly and voluntarily agreed upon by the parties without any
force, duress, or improper pressure being brought to bear upon the employee and
absent any other circumstances vitiating his consent; or second, it satisfactorily
appears that the employer and the employee dealt with each other on more or less
equal terms with no moral dominance exercised by the former or the latter.
In the context of the facts that: (1) the respondents were doing tasks
necessarily to Lynvils fishing business with positions ranging from captain of the
vessel to bodegero; (2) after the end of a trip, they will again be hired for another
trip with new contracts; and (3) this arrangement continued for more than ten
years, the clear intention is to go around the security of tenure of the respondents
as regular employees. And respondents are so by the express provisions of the
second paragraph of Article 280, thus: xxx Provided, That any employee who has
rendered at least one year of service, whether such service is continuous or broken,
shall be considered a regular employee with respect to the activity in which he is
employed and his employment shall continue while such activity exists.
Having found that respondents are regular employees who may be,
however, dismissed for cause as we have so found in this case, there is a need to
look into the procedural requirement of due process in Section 2, Rule XXIII, Book V
of the Rules Implementing the Labor Code. It is required that the employer furnish
the employee with two written notices: (1) a written notice served on the
employee specifying the ground or grounds for termination, and giving to said
employee reasonable opportunity within which to explain his side; and (2) a written
notice of termination served on the employee indicating that upon due
consideration of all the circumstances, grounds have been established to justify his
termination. In this case, it is clear that the employees were not given the final
written notices of dismissal.
The Court ruled that since employees were dismissed for just cause, they
were not entitled to separation pay and backwages. However, they were to be
granted nominal damages for failure of the employer to comply with statutory due
process.
Court has ruled that in labor cases, the corporate directors and officers are
solidarily liable with the corporation for the termination of employment of
employees done with malice or in bad faith. Indeed, moral damages are recoverable
when the dismissal of an employee is attended by bad faith or fraud or constitutes
an act oppressive to labor, or is done in a manner contrary to good morals, good
customs or public policy. We agree with the ruling of both the NLRC and the Court
of Appeals when they pronounced that there was no evidence on record that
indicates commission of bad faith on the part of De Borja. He is the general
manager of Lynvil, the one tasked with the supervision by the employees and the
operation of the business. However, there is no proof that he imposed on the
respondents the por viaje provision for purpose of effecting their summary
dismissal.
132. D.M. Consunji, Inc. v Jamin
GR 192514
Facts
DMCI, a construction company, hired Jamin as a laborer. Jamin became a helper
carpenter. Since hiring, Jamin's employment contract had been renewed a number
of times. His contract was terminated, however, due to the completion of the SM
Manila project. Jamin was not rehired again.
Jamin filed a complaint for illegal dismissal with monetary claims; that he was
terminated from employment without just and authorized cause at a time when he
was already 55 years old and had no independent source of livelihood. He claimed
to have rendered almost 31 years of service.
41

DMCI argued that it hired Jamin as a mere project employee. It alleged that it
submitted a report to DOLE everytime it terminated Jamin's services.
Compulsory Arbitration Ruling: LA dismissed Complaint. Jamin was a project
employee whose services had been terminated due to the completion of the
project where he was assigned. Every time DMCI rehired Jamin, it entered into a
contract of employment with him; and upon completion of the project, DMCI
served a notice of termination to him and a termination report to the DOLE regional
Office. Moreover, Jamin had to file an application if he wanted to be re-hired.
NLRC: Dismissed appeal, Affirmed LA ruling.
CA: Jamin is a regular employee, and he was illegally dismissed because the
termination was without valid cause and without due process. It noted that DMCI
failed to submit a report to the DOLE Regional Office everytime Jamin's
employment was terminated, as required by DOLE Policy Instructions No. 20. The
failure to submit the reports is an indication that Jamin was not a project employee.
It noted that DOLE D.O. No. 19, Series of 1993, which superseded DOLE P.I. No. 20
provides that the termination report is one of the indicators of project employment.
DMCI's MR denied.
Issues
The nature of Jamin's employment
Jamin is a regular employee. The Court affirmed the CA's findings which were based
on the following: 1) Jamin's repeated and successive engagements in DMCI's
construction projects, and 2) Jamin's performance of activities necessary or
desirable in DMCI's usual trade or business. [Emphasis mine]
In Liganza v. RBL Shipyard Corporation, the Court held that "[a]ssuming, without
granting[,] that [the] petitioner was initially hired for specific projects or
undertakings, the repeated re-hiring and continuing need for his services for over
eight (8) years have undeniably made him a regular employee.
The Liganza ruling is squarely applicable to this case, considering that:
1. DMCI repeatedly, continuously and successively hired Jamin
2. for almost 31 years, starting from Dec 17, 1968
3. for a total of 38 times -- 35 as shown by the schedule of projects submitted
by DMCI to the labor arbiter, and 3 more projects added by Jamin which he
claimed DMCI intentionally excluded in its report
A review of Jamin's employment contract
The contracts show that Jamin had been engaged as a project employee. However,
there was an almost unbroken string of Jamin's rehiring from Dec 17, 1968 - March
20, 1999. While the history of Jamins employment (schedule of projects) relied
upon by DMCI shows a gap of almost four years in his employment for the period
between July 28, 1980 (the supposed completion date of the Midtown Plaza
project) and June 13, 1984 (the start of the IRRI Dorm IV project), the gap was
caused by the companys omission of the three projects above mentioned.
To reiterate, Jamins employment history with DMCI stands out for his continuous,
repeated and successive rehiring in the companys construction projects. In all the
38 projects where DMCI engaged Jamins services, the tasks he performed as a
carpenter were indisputably necessary and desirable in DMCIs construction
business.
Is it necessary for a project employee to be a member of the work pool in order to
be considered regular?
No. In context, Jamin might not have been a member of a work pool as DMCI
insisted that it does not maintain a work pool, but his continuous rehiring and the
nature of his work unmistakably made him a regular employee.
Note: In Maraguinot, Jr. v. NLRC,the Court held that once a project or work pool
employee has been: (1) continuously, as opposed to intermittently, rehired by the
same employer for the same tasks or nature of tasks; and (2) these tasks are vital,
necessary and indispensable to the usual business or trade of the employer, then
the employee must be deemed a regular employee.

Вам также может понравиться